Health Assessment OLD CH

Ace your homework & exams now with Quizwiz!

Striae

(lineae albicantes) silvery white or pink scar tissue formed by stretching of abdominal skin as with pregnancy or obesity

Somatic Pain

(parietal) originates from the musculoskeletal tissues or body surface

Lesion DISCRETE

, distinct, individual lesions that remain separate (e.g., acrochordon or skin tags, acne).

Tinea Pedis (Ringworm of the Foot)

"Athlete's foot," a fungal infection, first appears as small vesicles between toes, on sides of feet, and on soles; grows scaly and hard. Found in chronically warm, moist feet: children after gymnasium activities, athletes, aging adults who cannot dry their feet well.

Pulse Amplitude: 0

Absent (Pulse amp)

Secondary Skin Lesions SCAR

After a skin lesion is repaired, normal tissue is lost and replaced with connective tissue (collagen). This is a permanent fibrotic change. Examples: healed area of surgery or injury, acne.

Ankyloglossia

Ankyloglossia

Interpret findings obtained during auscultation of the abdomen.

Auscultate Bowel Sounds and Vascular Sounds Depart from the usual examination sequence and auscultate the abdomen next. This is done because percussion and palpation can increase peristalsis, which would give a false interpretation of bowel sounds. Use the diaphragm endpiece because bowel sounds are relatively high-pitched. Hold the stethoscope lightly against the skin; pushing too hard may stimulate more bowel sounds (Fig. 22.11). Begin in the RLQ at the ileocecal valve area because bowel sounds normally are always present here. Bowel Sounds Note the character and frequency of bowel sounds. Although the origin of bowel sounds is not fully understood, they may originate from the movement of air and fluid within the stomach and large and small intestine. A wide range of normal sounds can occur. Normal bowel sounds are high-pitched, gurgling, cascading sounds, occurring irregularly anywhere from 5 to 30 times per minute. Do not bother to count them. In addition, because the sounds radiate widely over the abdomen, the gurgle you hear in the RLQ may originate in the stomach. Therefore listening in all four quadrants is not necessary.10 Just judge if they are present or are hypoactive or hyperactive. One type of hyperactive bowel sounds is fairly common: hyperperistalsis, when you feel your "stomach growling," termed borborygmus. Bowel sound interpretation is highly subjective and can vary widely among clinicians, and bowel sounds are likely not a reliable indicator of bowel function in many circumstances.5,19a For example, when assessing for the return of GI function after abdominal surgery, bowel sounds are less reliable than the passage of flatus and stool, as well as tolerance of oral intake.13 A perfectly "silent abdomen" is uncommon; you must listen for 5 minutes by your watch before deciding whether bowel sounds are completely absent. Two distinct patterns of abnormal bowel sounds may occur: 1. Hyperactive sounds are loud, high-pitched, rushing, tinkling sounds that signal increased motility. 2. Hypoactive or absent sounds follow abdominal surgery or with inflammation of the peritoneum Vascular Sounds As you listen to the abdomen, note the presence of any vascular sounds or bruits. Using firmer pressure, check over the aorta, renal arteries, iliac, and femoral arteries, especially in people with hypertension (Fig. 22.12). Usually no such sound is present. However, about 4% to 20% of healthy people (usually younger than 40 years) may have a normal bruit originating from the celiac artery.10 It is systolic, medium to low in pitch, and heard between the xiphoid process and the umbilicus. Note location, pitch, and timing of a vascular sound. A systolic bruit is a pulsatile blowing sound and occurs with stenosis, partial occlusion, or aneurysm of an artery. Venous hum and peritoneal friction rub are rare For safe practice, do NOT use auscultation of the abdomen for the correct placement of nasogastric tubes. Despite evidence showing that auscultation of an air bolus is not adequate to determine placement in the stomach or lung, you may see some nurses still practicing this method. Current evidence mandates confirming initial placement by chest x-ray and supports continuing assessment by measuring the external portion of the tube and testing the pH of stomach aspirates (pH less than 5.5 is acceptable). Ongoing visualization of gastric aspirates is also important to ensure that the tube has not migrated; fasting gastric secretions range from clear to green or brown. The auscultation method can wrongly suggest that the feeding tube is correctly placed in the stomach; serious harm or even fatality can result from administering tube-feeding material or medications into the lung.

carcinoma of the breast

Bloody nipple discharge that is unilateral and from a single duct requires further investigation. Although there was no palpable lump associated with the discharge shown here, mammography revealed a 1-cm, centrally located, ill-defined mass.

List the functions of the musculoskeletal system.

(1) for support to stand erect (2) for movement. The musculoskeletal system also functions (3) to encase and protect the inner vital organs (e.g., brain, spinal cord, heart) (4) to produce the red blood cells, white blood cells, and platelets in the bone marrow (hematopoiesis); (5) as a reservoir for storage of essential minerals such as calcium and phosphorus in the bones.

What is not a physiologic change for an aging adult?

CONSTIPATION

Star-Shaped Opacity—Cortical Cataract

Cortical cataract shows as asymmetric, radial, white spokes with black center. Through ophthalmoscope, black spokes are evident against the red reflex (not shown here). This forms in the outer cortex of lens, progressing faster than nuclear cataract.

Follicular Hyperkeratosis

Dry, bumpy skin associated with vitamin A and/or linoleic acid (essential fatty acid) deficiency. Linoleic acid deficiency may also result in eczematous skin, especially in infants.

Cyst

Encapsulated fluid-filled cavity in dermis or subcutaneous layer, tensely elevating skin. Examples: sebaceous cyst, wen.

Interpret findings obtained during inspection of the abdomen.

Inspect the Abdomen Contour Stand on the person's right side and look down on the abdomen. Then stoop or sit to gaze across the abdomen. Your head should be slightly higher than the abdomen. Determine the profile from the rib margin to the pubic bone. The contour describes the nutritional state and normally ranges from flat to rounded Scaphoid abdomen caves in. Protuberant abdomen indicates abdominal distention Symmetry Shine a light across the abdomen toward you or lengthwise across the person. The abdomen should be symmetric bilaterally (Fig. 22.8). Note any localized bulging, visible mass, or asymmetric shape. Even small bulges are highlighted by shadow. Step to the foot of the examination table to recheck symmetry. Bulges, masses. Hernia—Protrusion of abdominal viscera through abnormal opening in muscle wall Sister Mary Joseph nodule is a hard nodule in umbilicus that occurs with metastatic cancer of stomach, large intestine, ovary, or pancreas.10 Ask the person to take a deep breath to further highlight any change. The abdomen should stay smooth and symmetric. Or ask the person to perform a sit-up without pushing up with his or her hands. Note any localized bulging. Hernia or enlarged liver or spleen may show. Umbilicus Normally it is midline and inverted, with no sign of discoloration, inflammation, or hernia. It becomes everted and pushed upward with pregnancy. The umbilicus is a common site for piercings. The site should not be red or crusted. Everted with ascites or underlying mass Deeply sunken with obesity. Enlarged, everted with umbilical hernia. Bluish periumbilical color occurs (though rarely) with intraperitoneal bleeding (Cullen sign).10 Skin The surface is smooth and even, with homogeneous color. This is a good area to judge pigment because it is often protected from sun. Redness with localized inflammation. Jaundice (shows best in natural daylight). Skin glistening and taut with ascites. One common pigment change is striae (lineae albicantes)—silvery white, linear, jagged marks about 1 to 6 cm long (Fig. 22.9). They occur when elastic fibers in the reticular layer of the skin are broken after rapid or prolonged stretching as in pregnancy or excessive weight gain. Recent striae are pink or blue; then they turn silvery white. Striae also occur with ascites. Striae look purple-blue with Cushing syndrome (excess adrenocortical hormone causes the skin to be fragile and easily broken from normal stretching). Pigmented nevi (moles)—circumscribed brown macular or papular areas—are common on the abdomen. Unusual color or change in shape of mole Petechiae. Normally no lesions are present, although you may note well-healed surgical scars. If a scar is present, draw its location in the person's record, indicating the length in centimeters (Fig. 22.10). (NOTE: Infrequently a person may forget a past operation when providing the history. If you note a scar now, ask about it.) A surgical scar alerts you to the possible presence of underlying adhesions and excess fibrous tissue. Spider angiomas occur with portal hypertension or liver disease. Underlying adhesions are inflammatory bands that connect opposite sides of serous surfaces after trauma or surgery. Veins usually are not seen, but a fine venous network may be visible in thin persons. Prominent, dilated veins (caput medusae) occur with portal hypertension, cirrhosis, ascites, or vena caval obstruction. Veins are more visible with malnutrition as a result of thinned adipose tissue. Good skin turgor reflects adequate hydration. Gently pinch up a fold of skin; then release to note the immediate return of the skin to original position. Poor turgor occurs with dehydration, which often accompanies GI disease. Pulsation or Movement Normally you may see the pulsations from the aorta beneath the skin in the epigastric area, particularly in thin people with good muscle wall relaxation. Respiratory movement also shows in the abdomen, particularly in males. Finally, waves of peristalsis sometimes are visible in very thin people. They ripple slowly and obliquely across the abdomen. Marked pulsation of aorta occurs with widened pulse pressure (e.g., hypertension, aortic insufficiency, thyrotoxicosis) and aortic aneurysm. Marked visible peristalsis, together with a distended abdomen, indicates intestinal obstruction. Hair Distribution The pattern of pubic hair growth normally has a diamond shape in adult males and an inverted triangle shape in adult females Patterns alter with endocrine or hormone abnormalities, chronic liver disease. Demeanor A comfortable person is relaxed quietly on the examining table and has a benign facial expression and slow, even respirations. Restlessness and constant turning to find comfort occur with the colicky pain of gastroenteritis or bowel obstruction Absolute stillness, resisting any movement, occurs with the pain of peritonitis. Knees flexed up, facial grimacing, and rapid, uneven respirations also indicate pain.

Impetigo

Moist, thin-roofed vesicles with thin, erythematous base. Rupture to form erosions and thick, honey-colored crusts. Highly contagious bacterial infection of skin; most common in infants and children. Infection can spread to other body areas and other children and adults by direct contact.17

Ventricle

Muscular pumping chamber; thick walled

Pulse Amplitude: 2+

Normal (Pulse amp)

Decreased Tactile Fremitus

Occurs when anything obstructs transmission of vibrations (e.g., an obstructed bronchus, pleural effusion or thickening, pneumothorax, and emphysema). Any barrier that gets in the way of the sound and your palpating hand decreases fremitus.

Optic Atrophy (Disc Pallor)

Optic atrophy is a white or gray color of the disc as a result of partial or complete death of the optic nerve. This results in decreased visual acuity, decreased color vision, and decreased contrast sensitivity.

freckles (ephelides)—

Small, flat macules of brown melanin pigment that occur on sun-exposed skin

Normal Heart Sounds

Sounds S1 and S2

1) TRUE OR FALSE: The foramen ovale closes within 1 hour after birth.

T

What stage of pressure ulcer would you expect if you took a bite out of the apple and you're into the juicy meat of the apple?

Three

What is included in the Clinical Portrait of an Intestinal Obstruction? Select All That Apply

VOMITING ABDOMINAL TENDERNESS RESTLESSNESS

Explain the term capacitance vessels.

Veins are called capacitance vessels because of their ability to stretch. walls of the veins are thinner than those of the arteries. Veins have a larger diameter and are more distensible; they can expand and hold more blood when blood volume increases. this compensatory mechanism o reduce stress on the heart because of this ability to stretch, veins are called capacitance vessels

1) Lengthening of the bones occur in the:

a. Epiphyses

scaphoid

abnormally sunken abdominal wall, as with malnutrition or underweight

Lesion ANNULAR, or circular,

begins in center and spreads to periphery (e.g., tinea corporis or ringworm, tinea versicolor, pityriasis rosea).

Dysphagia

difficult swallowing

1) What type of heart sound is normal in children?

innocent murmurs

Anorexia

loss of appetite for food

Atherosclerosis

plaques of fatty deposits formed in the inner layer (intima) of the arteries

Apical Impulse

point of maximal impulse (PMI); pulsation created as the left ventricle rotates against the chest wall during systole, normally at the 5th left intercostal space in the midclavicular line; located at 4th-5th intercostal, at or inside the midclavicular line. Found using finger pads

Keratoses are

raised, thickened areas of pigmentation that look crusted, scaly, and warty. One type, seborrheic keratosis, looks dark, greasy, and "stuck on" (Fig. 13.26). They develop mostly on the trunk but also on the face and hands and on both unexposed and sun-exposed areas. They do not become cancerous.

Elevation

raising a body part

Tachycardia

rapid heart rate; greater than 100 BPM

List three conditions that result in cardiac murmurs

1. Velocity of blood increases (flow murmur) (e.g., in exercise, thyrotoxicosis) 2. Viscosity of blood decreases (e.g., in anemia) 3. Structural defects in the valves (a stenotic or narrowed valve, an incompetent or regurgitant valve) or unusual openings occur in the chambers (dilated chamber, septal defect) Murmurs - Gentle, blowing, swooshing sound that can be heard on chest wall - Conditions that create turbulent blood flow and collision currents •Physical activity or exercise •Pregnancy •Fever •Not having enough healthy red blood cells to carry adequate oxygen to your body tissues (anemia) •An excessive amount of thyroid hormone in your body (hyperthyroidism) •Phases of rapid growth, such as adolescence •Holes in the heart or cardiac shunts. •Heart valve abnormalities •Valve calcification •Endocarditis •Rheumatic fever

Pericordium

Anterior wall that overlies the heart and the great vessels.

What a day keeps the doctor away?

Apple

Which medication causes an increased risk of falling for the aging adult?

BENZODIAZEPINES

°F to °C

Deduct 32, then multiply by 5, then divide by 9

Osteoarthritis (OA)

Different from RA, OA is characterized by hard, nontender, noninflammatory nodules, 2 to 3 mm or more. These osteophytes (bony overgrowths) of the DIP joints are called Heberden nodes. Those of the PIP joints are called Bouchard nodes and are less common.

°C to °F

Divide by 5, then multiply by 9, then add 32

Heart Anatomy

Extends from left 2nd ICS to left 5th ICS; extends horizontally from right edge of sternum to left midclavicular line.

When performing the abdomen assessment I begin with which skill?

INSPECT

Pulse Amplitude: 3+

Increased (Pulse amp)

Discuss the developmental care regarding the musculoskeletal system.

Infants and Children - By 3 months' gestation, the fetus has formed a "scale model" of the skeleton that is made up of cartilage. During succeeding months in utero, the cartilage ossifies into true bone and starts to grow. Bone growth continues after birth—rapidly during infancy and then steadily during childhood—until adolescence, when both boys and girls undergo a rapid growth spurt. Long bones grow in two dimensions. They increase in width or diameter by deposition of new bony tissue around the shafts. Lengthening occurs at the epiphyses, or growth plates. These specialized growth centers are transverse discs located at the ends of long bone. Any trauma or infection at this location puts the growing child at risk for bone deformity. This longitudinal growth continues until closure of the epiphyses; the last closure occurs at about age 20 years. Skeletal contour changes are apparent at the vertebral column. At birth the spine has a single C-shaped curve. At 3 to 4 months, raising the baby's head from prone position develops the anterior curve in the cervical neck region. From 1 year to 18 months, standing erect develops the anterior curve in the lumbar region. Although the skeleton contributes to linear growth, muscles and fat are significant for weight increase. Individual muscle fibers grow throughout childhood, but growth is marked during the adolescent growth spurt. Then muscles respond to increased secretion of growth hormone to adrenal androgens and in boys to further stimulation by testosterone. Muscles vary in size and strength in different people. This is because of genetic programming, nutrition, and exercise. All through life muscles increase with use and atrophy with disuse. The Pregnant Woman - Increased levels of circulating hormones (estrogen, relaxin from the corpus luteum, and corticosteroids) cause increased mobility in the joints. Increased mobility in the sacroiliac, sacrococcygeal, and symphysis pubis joints in the pelvis contributes to the noticeable changes in maternal posture. The most characteristic change is progressive lordosis, which compensates for the enlarging fetus; otherwise the center of balance would shift forward. Lordosis compensates by shifting the weight farther back on the lower extremities. This shift in balance in turn creates strain on the low back muscles, which in some women is felt as low back pain during late pregnancy. Anterior flexion of the neck and slumping of the shoulder girdle are other postural changes that compensate for the lordosis. These upper-back changes may put pressure on the ulnar and median nerves during the third trimester. Nerve pressure creates aching, numbness, and weakness in the upper extremities in some women. The Aging Adult - Peak bone mass or bone mineral density (BMD) is reached in the early to late 20s for Caucasians, with females reaching their peaks significantly earlier than males.18 After that bone remodeling occurs, which is the cyclic process of bone resorption and deposition responsible for skeletal maintenance at sites that need repair or replacement. The standard pattern is for bone resorption to equal bone replacement. When bone resorption (loss of bone matrix) occurs more rapidly, the net effect is a gradual loss of bone density or osteoporosis. Osteoporosis is a disease involving the loss of mineralized bone mass and leading to porous bone and thus the risk of fractures. Although aging women have a greater amount of bone loss compared with aging men, decreased levels of estrogen in both sexes are partly responsible because osteoblasts that form new bone have estrogen receptors. Resistance and weight-bearing exercise may increase bone density at the hip and lumbar spine, but exercise needs to be regular and high intensity.2 Further, its effects may be lost once exercise is stopped. (See Health Teaching on p. 608 for more lifestyle changes to retard osteoporosis.) Postural changes are evident with aging. Long bones do not shorten with age. Decreased height of 3 to 5 cm occurs with shortening of the vertebral column, caused by loss of water content and thinning of the intervertebral discs and by a decrease in the height of individual vertebrae from osteoporosis. A progressive decrease in height is not significant until 60 years. A greater decrease occurs in the 70s and 80s as a result of osteoporotic collapse of the vertebrae. Other postural changes are an increase in the thoracic curve (kyphosis), a backward head tilt to compensate for the kyphosis, and a slight flexion of hips and knees. The distribution of subcutaneous fat changes through life. Usually men and women gain weight in their 40s and 50s. They begin to lose fat in the face and deposit it in the abdomen and hips. In the 80s and 90s, fat further decreases in the periphery, especially noticeable in the forearms, and increases over the abdomen and hips. Loss of subcutaneous fat leaves bony prominences more marked (e.g., tips of vertebrae, ribs, iliac crests) and body hollows deeper (e.g., cheeks, axillae). An absolute loss in muscle mass occurs; some muscles decrease in size, and some atrophy, producing weakness. Lifestyle affects musculoskeletal changes; a sedentary lifestyle hastens musculoskeletal changes of aging. However, physical exercise increases skeletal mass and helps prevent or delay osteoporosis. Physical activity delays or prevents bone loss in postmenopausal women in a dose-dependent manner23 (see Patient Teaching on p. 608).

Arteriovenous Crossing (Nicking)

Inset shows arteriovenous crossing with interruption of blood flow. When vein is occluded, it dilates distal to crossing. This person also has disc edema and hard exudates in a macular star pattern that occur with acutely elevated (malignant) hypertension. With hypertension, the arteriole wall thickens and becomes opaque so that no blood is seen inside it (silver-wire arteries).

Inspection, Auscultation, Percussion, Palpation Feces

Inspection. Localized distention. Auscultation. Normal bowel sounds. Percussion. Tympany predominates. Scattered dullness over fecal mass. Palpation. Plastic-like or ropelike mass with feces in intestines.

Use the correct sequence of examining techniques.

Inspection: Abdomen flat, symmetric, with no apparent masses. Skin smooth with no striae, scars, or lesions. Auscultation: Bowel sounds present, no bruits. Percussion: Tympany predominates in all 4 quadrants. Palpation: Abdomen soft, no organomegaly, no masses, no tenderness. Inspection Contour Symmetry Umbilicus Skin Pulsation or movement Hair distribution Demeanor 2. Auscultation Bowel sounds Note any vascular sounds 3. Percussion Percuss all four quadrants 4. Palpation Light palpation in all four quadrants Deeper palpation in all four quadrants Palpate for liver, spleen, kidneys

Periorbital Edema

Lids are swollen and puffy. Lid tissues are loosely connected, so excess fluid is easily apparent. This occurs with local infections; crying; trauma; and systemic conditions such as congestive heart failure, renal failure, allergy, hypothyroidism (myxedema).

Describe the ways of documenting clinical findings from a breast examination.

Location Size Shape Consistency Movable Distinctness Nipple Note skin over lump Tenderness Lymphadenopathy Document pain, lump, discharge, rash, swelling, trauma, history of surgery, pregnancy, arm movement with dimpling or retraction.

Sebaceous Cyst

Location is commonly behind lobule in the postauricular fold. A nodule with central black punctum indicates blocked sebaceous gland. It is filled with waxy sebaceous material and painful if it becomes infected. Often are multiple.

Describe the function of the lymph nodes.

Lymph Nodes filter lymph from the body and drain to the kidney for expulsion.

Lymphedema Precautions

Means that absolutely no blood pressure or blood draws can be done on that arm.

Musculoskeletal Signs Associated With Malnutrition AND Nutrient Deficiency

Pain in calves, thighs Thiamine Osteomalacia Vitamin D, calcium Rickets Vitamin D, calcium Joint pain Vitamin C Muscle wasting Protein, carbohydrate, fat

Papule vs Plaque

Papule Something you can feel (i.e., solid, elevated, circumscribed, less than 1 cm diameter) caused by superficial thickening in epidermis. Examples: elevated nevus (mole), lichen planus, molluscum, wart (verruca). Plaque Papules coalesce to form surface elevation wider than 1 cm. A plateaulike, disk-shaped lesion. Examples: psoriasis, lichen planus.

Geographic Tongue (Migratory Glossitis)

Pattern of normal coating interspersed with bright red, shiny, circular bald areas caused by atrophy of the filiform papillae, with raised pearly borders. Pattern resembles a map and changes with time. Not significant, and its cause is not known.

Tinea Corporis (Ringworm of the Body)

Scales—hyperpigmented in whites, depigmented in dark-skinned people; on chest, abdomen, back of arms forming multiple circular lesions with clear centers.

Secondary Skin Lesions EROSION

Scooped out but shallow depression. Superficial; epidermis lost; moist but no bleeding; heals without scar because erosion does not extend into dermis.

Herpes Zoster (Shingles)

Small, grouped vesicles emerge along route of cutaneous sensory nerve, then pustules, then crusts. Caused by the varicella zoster virus (VZV), a reactivation of the dormant virus of chickenpox. Acute appearance, unilateral, does not cross midline. Commonly on trunk; can be anywhere. If on ophthalmic branch of cranial nerve V, it poses risk to eye. Most common in adults older than 50 years. Pain is often severe and long-lasting in aging adults, called postherpetic neuralgia.

Heave/Lift

Sustained forceful thrusting of the ventricle during systole. Occurs due to increased workload.

Describe the tympanic membrane and its anatomic landmarks.

The TM separates the external and middle ear· it is tilted obliquely to ear canal facing downward and somewhat forward· it is translucent with pearly gray color and a prominent "cone of light" which is the reflection of the otoscope light - at 5 in right ear and at 7 in left ear· it is oval and slightly concave cone of lightmalluspars flaccida: small, slack superior section of membranepars tensa: thicker and taut

Describe the location and function of the various joints in the body.

The temporomandibular joint (TMJ) is the articulation of the mandible and the temporal bone. The TMJ permits jaw function for speaking and chewing. The joint allows three motions: (1) hinge action to open and close the jaws; (2) gliding action for protrusion and retraction; and (3) gliding for side-to-side movement of the lower jaw. The shoulder girdle is a belt of three large bones (humerus, scapula and clavicle), joints, and muscles. The glenohumeral joint is the articulation of the humerus with the glenoid fossa of the scapula. Note the humeral head's articular surface is much greater than the glenoid's articular surface. This ball-and-socket action allows great mobility of the arm on many axes, more than any other joint. The joint is enclosed by a group of four powerful muscles and tendons that support and stabilize it. Together these are called the rotator cuff. The four muscles are the SITS muscles (supraspinatus, infraspinatus, teres minor, and subscapularis). These form a cover around the head of the humerus. They rotate the arm laterally and stabilize the head of the humerus against the shallow glenoid fossa of the scapula. The large subacromial bursa helps during abduction of the arm so that the greater tubercle of the humerus moves easily under the acromion process of the scapula. Of the body's 206 bones, over half are in the hands and feet. The wrist, or radiocarpal joint, is the articulation of the distal radius (on the thumb side) and a row of 8 carpal bones. Its condyloid action permits movement in two planes at right angles: flexion and extension, and side-to-side deviation. You can feel the groove of this joint on the dorsum of the wrist. The midcarpal joint is the articulation between the two parallel rows of carpal bones. It allows flexion, extension, and some rotation. The metacarpophalangeal (MCP) and the interphalangeal joints (DIP and PIP) permit finger flexion and extension. The flexor tendons of the wrist and hand are enclosed in synovial sheaths. The elbow joint contains the three bony articulations of the humerus, radius, and ulna of the forearm. Its hinge action moves the forearm (radius and ulna) on one plane, allowing flexion and extension. The muscles are the biceps and brachioradialis for flexion, and the triceps and brachialis for extension. The olecranon bursa lies between the olecranon process and the skin. The hip joint is the articulation between the cup-shaped acetabulum and the head of the femur (Fig. 23.11). As in the shoulder, ball-and-socket action permits a wide range of motion (ROM) on many axes. The hip has somewhat less ROM than the shoulder, but it has more stability as befits its weight-bearing function. Hip stability is the result of powerful muscles that spread over the joint, a strong fibrous articular capsule, and the very deep insertion of the head of the femur. The muscles include the anterior flexor (iliopsoas), the posterior extensor (gluteus maximus), adductor muscles that swing the thigh toward the midline, and abductor muscles that swing it away. Three bursae facilitate movement. The knee joint is the articulation of three bones—the femur, the tibia, and the patella (kneecap)—in one common articular cavity (Fig. 23.12). It is the largest joint in the body and is complex. It is a hinge joint, permitting flexion and extension of the lower leg on a single plane. There is no overlying fat or muscle; only the ligaments hold the tibia and femur in place, making the knee vulnerable to injury. The ankle, or tibiotalar joint, is the articulation of the tibia, fibula, and talus (Fig. 23.14). It is a hinge joint, limited to flexion (dorsiflexion) and extension (plantar flexion) on one plane. Landmarks are two bony prominences on either side: the medial malleolus and the lateral malleolus. Strong, tight medial and lateral ligaments extend from each malleolus onto the foot. These help the lateral stability of the ankle joint, although they may be torn in eversion or inversion sprains of the ankle. The calcaneus (heel) is under the talus and points posteriorly.

Intraductal Papilloma

These are discrete benign tumors that arise in a single or multiple papillary duct(s). May have serous or serosanguineous discharge. Often there is a palpable nodule in underlying duct (highlighted here). Most common in women ages 40 to 60 years. Most are benign, although multiple papillomas have a higher risk of subsequent cancer than do solitary ones. Requires core needle biopsy and possible excision.

Black Hairy Tongue

This is not really hair but rather the elongation of filiform papillae and painless overgrowth of mycelial threads of fungus infection on the tongue. Color varies from black-brown to yellow. It occurs after use of antibiotics, which inhibit normal bacteria and allow proliferation of fungus, and with heavy smoking.

Mastitis

This is uncommon; an inflammatory mass before abscess formation. Usually occurs in single quadrant. Area is red, swollen, tender, very hot, and hard, here forming outward from areola upper edge in right breast. The woman also has a headache, malaise, fever, chills, sweating, increased pulse, flulike symptoms. May occur during first 4 months of lactation from infection or from stasis from plugged duct. Treat with rest, local heat to area, antibiotics, and frequent nursing to keep breast as empty as possible. Must not wean now, or the breast will become engorged, and the pain will increase. Mother's antibiotic not harmful to infant. Usually resolves in 2 to 3 days.

What stage of pressure ulcer would you expect if the apple were peeled carefully so just the outside layer was missing?

Two

1) Which statement by the nurse is correct?

a. "The veins carry unoxygenated blood and wastes back to the heart."

1) Normal Heart Sounds may be documented as?

a. S1, S2

Hepatomegaly

abnormal enlargement of liver

Peritonitis

inflammation of peritoneum

Cholecystitis

inflammation of the gallbladder

Mitral regurgitation

mitral insufficiency; incompetent mitral valve allows regurgitation of blood back into left atrium during systole

Bradycardia

slow heart rate; less than 50 BPM

Diastole

the heart's filling phase Relaxation of the ventricles, known as filling.

Aortic Valve aka aortic semilunar valve

the left semilunar valve separating the left ventricle and the aorta Location: 2nd right interspace

Endocardium

thin layer of endothelial tissue that lines the inner surface of the heart chambers and valves Heart's innermost layer

•What is Alli's Test?

•Used to check for hip dislocation, lengths of legs compared

Describe the components of the physical examination of the known survivor of intimate partner violence (IPV) or elder abuse and use correct forensic terminology.

◦Gathering of subjective data ◦Use of open-ended questions to start the conversation ◦Interview the individual separately from the perpetrator. ◦Listen for cues which may indicate a pattern or responses that don't match the "physical" injury that is present. ◦Be aware of state laws and requirement to report. Assessment of Older Adult Abuse and Neglect ◦Older adult as a vulnerable population ◦Recommended routine screening by multiple agencies but no specific tool specified ◦Assessment of abuse or neglect in cognitively challenged persons is complicated. ◦Older adult abuse suspicion index ◦Validated in primary care ◦For use with cognitively intact patients ◦Includes 6 questions with 5 questions asked of the patient and the last question answered by the physician

Describe the pleurae and their function.

- Thin, slippery pleura form an envelope between the lungs and chest wall - Outside layer visceral; inside layer parietal - Function: form cushion for lungs - Pleural cavity has negative pressure or vacuum which holds the lungs tightly against chest wall - Helps lungs slide smoothly Serous membranes that form an envelope between lungs and chest wall Visceral Pleura- lines outside of lungs, dipping into fissures Parietal Pleura- lines inside of chest wall and diaphragm Pleural cavity- inside of envelope, potential space, filled with only few ml of lubricating fluid, vacuum (neg. pressure) Costodiaphragmatic recess- pleurae extend 3 cm below level of lungs, this is a "potential space" o Potential space exists for expansion, but it has fluid sitting in it so it has potential for infection.

Right lower quadrant include what organs

-cecum -appendix -right ovary and tube -right ureter -right spermatic cord

Auscultation

-done prior to percussion and palpation bc they can increase peristalsis and give false interpretation of bowel sounds -begin in RLQ at ileocecal valve bc bowel sounds are usu present here -note character and frequency -silent abdomen is uncommon; decreased motility or bowel obstruction. listen for 5 min to be sure.

Contour types of the stomach

-flat (normal) -rounded -scaphoid (malnourished or dehydrated) -protuberant (pregnancy, hernia)

Exam Sequence

-inspect -auscultate -percuss and palpate (very rarely)

Right upper quadrant include what organs

-liver -gallbladder -duodenum -head of pancreas -right kidney and adrenal -hepatic flexure of colon -part of ascending and transverse colon

Describe developmental care as well as cross-cultural and gender considerations regarding pain.

-not a normal process of aging.Gender- social expectationsCultural- misunderstanding, misinterpretation.

Dysfunction of the musculoskeletal system that result in or from disease or disability.

-osteoporosis -osteopenia -sarcopenia -rheumatoid arthritis (autoimmune)- osteoarthritis (from overuse)

Lower left quadrant include what organs

-part of ascending colon -sigmoid colon -left ovary and tube -left ureter -left spermatic cord

Left upper quadrant include what organs

-stomach -spleen -left lobe of liver -body of pancreas -left kidney and adrenal -splenic flexure of colon -part of transverse and descending colon

Blood flow through heart

-superior/inferior vena cava -right atrium -tricuspid valve -right ventricle -pulmonic valve (pulmonary semilunar valve) -pulmonary trunck->lung tissue-> -pulmonary veins -left atrium -mitral valve (bicuspid) -left ventricle -aortic semilunar valve -aorta -systemic circulation

**************** Use measures that will enhance abdominal wall relaxation.

-the person should have emptied bladder, saving a urine specimen if needed -keep room warm to avoid chilling and tensing of muscle -position patient in supine position, head on pillow, knees bent or on pillow, arms at the sides or across chest -to avoid abdominal tensing warm the end -piece of the stethoscope being used along with your hands .-inquire about painful areas, examine such an area last to avoid any muscle guarding -finally learn to use distraction, breathing exercises, emotive imagery, use a soothing voice, and engage in conversation

Flow of blood through the heart

....

List the anatomic landmarks of the external ear.

- External ear is called AURICLE or PINNA (consists of movable cartilage and skin) -its characteristic shape serves to tunnel sound waves into its opening; the EXTERNAL AUDITORYCANAL -The canal terminates at the EARDRUM or TYMPANIC MEMBRANE -Helix -Tragus -Lobe External Auditory Meatus, Tragus, Pinna,

Describe the most important points about the health history for the respiratory system

-Cough -Shortness of breath -Chest pain with breathing -History or respiratory infections -Smoking history -Environmental exposure -Self-care behaviours

The manubriosternal angle is also called ...? Why is it useful for landmarks?

... angle of Louis. Because it is continuous with the second rib

************ Identify pertinent topics that must be reviewed during the abdominal portion of the interview.

1. Appetite • Any change in appetite? • Any change in weight? Anorexia is a loss of appetite from GI disease as a side effect to some medications, with pregnancy, or with mental health disorders 2. Dysphagia • Any difficulty in swallowing? Dysphagia occurs with disorders of the throat or esophagus, such as thrush (candida infection), neurologic changes (e.g., stroke), or obstruction (e.g., solid mass or tumor). 3. Food intolerance • Are there any foods you cannot eat? Food intolerance (e.g., lactase deficiency resulting in bloating or excessive gas after taking milk products). Pyrosis (heartburn), a burning sensation in esophagus and stomach from reflux of gastric acid. Eructation (belching). 4. Abdominal pain • Any abdominal pain? 5. Nausea/vomiting • Any nausea or vomiting? 6. Bowel habits • How often do you have a bowel movement? 7. Past abdominal history • Any history of GI problems: ulcer, gallbladder disease, hepatitis/jaundice, appendicitis, colitis, hernia? 8. Medications • Which medications are you taking currently? 9. Nutritional assessment • Now I would like to ask you about your diet. Please tell me all the food you ate yesterday, starting with breakfast. • Which fresh food markets are located in your neighborhood? Additional History for Infants and Children 1. Are you breastfeeding or bottle-feeding the baby? If bottle-feeding, how does baby tolerate the formula? 2. Which table foods have you introduced? How does the infant tolerate the food? 3. How often does your toddler/child eat? Does he or she eat regular meals? How do you feel about your child's eating problems? 4. Does your child have constipation? How long? 5. Does the child have abdominal pain? Please describe what you have noticed and when it started. 6. For the overweight child: How long has weight been a problem? Additional History for Adolescents 1. What do you eat at regular meals? Do you eat breakfast? What do you eat for snacks? 2. What is your exercise pattern? 3. If weight is less than body requirements: How much have you lost? By diet, exercise, or how? Additional History for the Aging Adult 1. How do you acquire your groceries and prepare your meals? 2. Do you eat alone or share meals with others? 3. Please tell me all that you had to eat yesterday, starting with breakfast. 4. How often do your bowels move?

How many degrees is the normal costal angle?

90 degrees

Oral temp

97.6-99.6 98.6 Use the oral route when the child is old enough to keep his or her mouth closed. This is usually at age 5 or 6 years, although some 4-year-old children can cooperate.

Epulis

A benign nontender, fibrous nodule of the gum seen emerging between the teeth; an overgrowth of vascular granulation tissue.

Pectus Carinatum

A forward protrusion of the sternum, with ribs sloping back at either side and vertical depressions along costochondral junctions (pigeon breast). Less common than pectus excavatum, this minor deformity requires no treatment. If severe, surgery may be indicated.

Murmur

A swishing sound caused by turbulent blood flow through the heart valves or great vessels. (like listening through conch shell)

Pterygium

A triangular opaque wing of bulbar conjunctiva overgrows toward the center of the cornea. It looks membranous, translucent, and yellow to white; usually invades from nasal side; and may obstruct vision as it covers pupil. It occurs usually from chronic exposure to a hot, dry, sandy climate, which stimulates the growth of a pinguecula (see p. 300) into a pterygium.

Identify the structures viewed through the ophthalmoscope.

A. The optic disc, the retinal vessels, the general background, and the macula

1) A mother of a newborn is concerned about her 5 month old's hip. When performing the __________ test the nurse notices the leg lengths are different.

ALLIS

Nipple Fixation

Asymmetry, distortion, or decreased mobility with the elevated arm maneuver. As cancer becomes invasive, the fibrosis fixes the breast to the underlying pectoral muscles. Here note that the right breast is held against the chest wall.

Coxa Plana (Legg-Calvé-Perthes Syndrome)

Avascular necrosis of the femoral head, occurring primarily in males between 3 and 12 years of age, with peak at age 6 years. In initial inflammatory stage interruption of blood supply to femoral epiphysis occurs, halting growth. Revascularization and healing occur later, but significant residual deformity and dysfunction may be present.

((((((((((((((((((((((())))))))))))))))))))))))) Identify significant breast anatomy and breast lymphatic system.

BREASTS Mammary glands- present in both females & males Male breasts- rudimentary throughout life Female breasts- accessory reproductive organs whose function is to produce milk for nourishing newborn Extensive lymphatic drainage Surface Anatomy Location of breasts on chest wall Axillary tail of Spence •Upper LEFT quadrant(MOST breast cancer here and upper outer quadrant) Nipple and areola Internal Anatomy Glandular tissue, which contains Lobes, that contain Lobules, that contain Alveoli (produce milk) Each lobe drains into a Lactiferous duct o Ducts converge toward nipple where they form ampullae/lactiverous sinuses that store milk Fibrous tissue that support breast o Suspensory ligaments, aka Cooper's ligaments Adipose tissue o Lobes are embedded in adipose tissue Four quadrants of breast (see image) Upper LEFT quadrant is site of most breast tumors Next most common in nipple area The breast is located between the second and sixth rib anterior to the pectoralis muscle. LYMPHATIC >75% of lymph drains into the ipsilateral axillary nodes. Their are four groups. The Pectoral, Sub-scapular, and Lateral drain into the Central Axillary node. Lymph o Most of lymph, more than 75%, drains into ipsilateral (same side) axillary nodes Axillary nodes- o Pectoral/Anterior o Subscapular/ Posterior o Lateral All three drain into Central Axillary Nodes From the central axillary nodes, drainage flows UP to: o Infraclavicular and supraclavicular nodes Smaller amount of lymphatic drainage flows directly up to infraclavicular group, deep into chest, or into abdomen, or directly across to opposite breast

Discuss the significance of a "barrel chest"

Barrel chest has horizontal ribs and costal angle >90 degrees. Note equal anteroposterior-to-transverse diameter and that ribs are horizontal instead of the normal downward slope. This is associated with normal aging and also with chronic emphysema and asthma as a result of hyperinflation of lungs.

What is pulsus paradoxus?

Beats have weaker amplitude with inspiration and stronger amplitude with expiration. The rhythm is irregular and coupled in pulsus bigeminus; every other beat comes early or premature. A weak, thready pulse may result in a deficiency of arterial blood to a body part. The rhythm is regular in pulsus alternans, but the force varies with alternating beats of large and small amplitudes.

Flexion

Bending a body part

Gynecomastia

Benign enlargement of male breast that occurs when estrogen concentration exceeds testosterone levels. It is a mobile disk of tissue located centrally under the nipple-areola. At puberty it is usually mild and transient. In older men it is bilateral, tender, and firm but not as hard as breast cancer. Gynecomastia occurs with Cushing syndrome, liver cirrhosis (because estrogens cannot be metabolized), adrenal disease, hyperthyroidism, and numerous drugs: alcohol and marijuana; estrogen treatment for prostate cancer; antibiotics (metronidazole, isoniazid); spironolactone.

Fibroadenoma

Benign mass, most commonly self-detected in late adolescence and early adulthood. Solitary nontender mass that is solid, firm, rubbery, and elastic. Round, oval, or lobulated; 1 to 5 cm. Freely movable, slippery; fingers slide it easily through tissue. Usually no axillary lymphadenopathy but frequently painful. Diagnose by palpation, ultrasound, and needle biopsy. Because of risk of deformity of surgery to a growing breast, excisional surgery is reserved for masses >5 cm; for continuously enlarging, well-circumscribed, multiple masses; or with suspicious ultrasound findings.

Gums Signs Associated With Malnutrition AND Nutrient Deficiency

Bleeding Vitamin C

Hyphema

Blood in the anterior chamber is a serious result of herpes zoster infection. Also occurs with blunt trauma (a fist or a baseball) or spontaneous hemorrhage. Suspect scleral rupture or major intraocular trauma. Note that gravity settles blood in front of iris.

Pulse Amplitude: 4+

Bounding (Pulse amp)

1) The nurse is reviewing the blood supply to the arm. What major artery supplies blood to the arm? _____________________

Brachial

Major arteries of the Arm

Brachial, radial, and ulnar

Acute Tonsillitis and Pharyngitis

Bright red throat; swollen tonsils; white or yellow exudate on tonsils and pharynx; swollen uvula; and enlarged, tender anterior cervical and tonsillar nodes. Accompanied by severe sore throat, painful swallowing, fever >101° F of sudden onset. Bacterial infections may have absence of cough. With severe symptoms (listed above) or sore throat lasting >3-5 days, consider streptococcal infection and confirm with rapid antigen testing or throat culture. Treat positive tests with antibiotics. Untreated GAS pharyngitis may produce peritonsillar abscess, lymphadenitis, or acute rheumatic fever (although this is now rare in the United States).

Nails Signs Associated With Malnutrition AND Nutrient Deficiency

Brittle, ridged, or spoon-shaped (koilonychia) Iron Splinter hemorrhages Vitamin C

Oral Kaposi Sarcoma

Bruiselike, dark red or violet, confluent macule, usually on the hard palate, may be on soft palate or gingival margin. Oral lesions may be among the earliest lesions to develop with AIDS.

Salivary Glands

Can be blocked by salivary stones

Cardiac output equation

Cardiac Output= Heart Rate x Stroke Volume or CO=SV x R

Describe the carotid artery pulse.

Carotid Artery Pulse is found in the neck and is closely related to the heart rhythm. Closes to the heart strongest pulse and closes related to the pulse from the heart Its timing closely coincides with ventricular systole.located in the groove between the trachea and the sternomastoid muscle, medial to and alongside that muscle. Note the characteristics of its waveforma smooth rapid upstroke, a summit that is rounded and smooth, and a downstroke that is more gradual and has a dicrotic notch caused by closure of the aortic valve

Arteries

Carries oxygenated nutrient-rich blood to capillaries. High pressure system, thick layered walls, pulse is caused which is the force of blood against arterial walls. -carry oxygenated blood to body tissues -travels away from heart -high pressure system -walls are tough, strong and tense; withstand pressure demands

Veins

Carry deoxygenated nutrient-depleted waste-laden blood from tissues back to the heart. No force that propels blood flow, low pressure system. -drain deoxygenated blood, CO2 and waste products from tissues back to the heart -lower pressure system

Pericardial Friction Rub

Caused by inflammation of the pericardial sac; best heard using diaphragm over the 3rd ICS to the left sternum. High pitched, scratchy, scraping sound. Commonly heard during the first week after a myocardial infarction.

Physiologic Murmur

Caused by temporary increase in blood flow. Can occur with anemia, pregnancy, fever, and hyperthyroidism.

Telangiectasia

Caused by vascular dilation; permanently enlarged and dilated blood vessels that are visible on the skin surface.

Major veins of the Arm

Cephalic, Basilic, Median Cuital

Leukoplakia

Chalky white, thick, raised patch with well-defined borders. The lesion is firmly attached and does not scrape off. It may occur on the lateral edges of tongue. It is caused by chronic irritation of smoking and alcohol use. Lesions are precancerous; must refer to specialist. (Here the lesion is associated with squamous carcinoma.)

Incorporate health promotion concepts when performing an assessment of the peripheral vascular system.

Checking your feet every day. If you are unable to see the bottoms of your feet, use a mirror or ask someone to help you. Dry feet carefully after a shower or bath. Examine each foot for red spots or sensitive areas, discoloration of skin or nails, ingrown nails, pain, cuts, swelling, or blisters. Keep toenails trimmed, straight across, and filed at the edges with an emery board or nail file. Nail polish... do's and don'ts. Chipped nail polish may support the growth of larger numbers of organisms on nails. This is especially important for those already at risk for infection. Do not use nail polish to cover up discolored nails. Nail polish locks out moisture and keeps the nail bed from being able to 'breathe'. Keeping blood flowing to your feet. Walking is a great way to exercise your feet or try foot exercises such as: Sitting down and rotating your ankles in one direction, then the other, or try writing the alphabet from A to Z! In bare feet curl your toes and spread them out. When an individual is not able to walk, putting the feet up when sitting or lying down, stretching, wiggling toes, or having a gentle foot massage or warm foot bath is a great alternative. Wearing shoes that fit and are comfortable. The best time to measure feet is toward the end of the day, when feet tend to be the largest. Individuals often have one foot that is larger than the other. They should fit their shoes to the larger foot. Select shoes that are shaped like one's feet. The ball of the foot should fit comfortably into the widest part of the shoe. Toes should not be crowded. For women, low-heeled shoes are safer and less damaging than high-heeled shoes. Keeping skin soft and smooth. Use mild soap. Be careful about adding oils to bath water. They can make your feet and the bathtub both very slippery. A thin coat of skin lotion over the tops and bottoms of one's feet help to keep skin soft and smooth.

Dupuytren Contracture

Chronic hyperplasia of the palmar fascia causes flexion contractures of the digits, first in the 4th digit, then the 5th digit, and then the 3rd digit. Note the bands that extend from the midpalm to the digits and the puckering of palmar skin. Common in men older than 40 years and is usually bilateral. It occurs with diabetes, epilepsy, and alcoholic liver disease and as an inherited trait. The contracture is painless but impairs hand function.

Secondary Skin Lesions SCALE

Compact, desiccated flakes of skin, dry or greasy, silvery or white, from shedding of dead excess keratin cells. Examples: after scarlet fever or drug reaction (laminated sheets), psoriasis (silver, micalike), seborrheic dermatitis (yellow, greasy), eczema, ichthyosis (large, adherent, laminated), dry skin.

Relate the structure and functions of the lymph vessels.

Completely separate vessel system, which retrieves excess fluid and plasma proteins from the interstitial spaces and returns it to the bloodstream Functions of lymphatic system are to Øconserve fluid and plasma proteins that leak out of capillaries. Øform a major part of immune system that defends body against disease. Øabsorb lipids from intestinal tract. Lymph nodes are small oval clumps of lymphatic tissue located at intervals along vessels. ØSuperficial groups of nodes are accessible to inspection and palpation giving clues to status of lymphatic system. ØPressure gradient difference between arteriolar and venous end of capillary beds ØHydrostatic and colloid osmotic pressure structure- thin but is valved, it can push but it can not fall back the blood that is function: get fluid to the body... the infection possy

Lymphatic System

Component of circulatory system; composed of lymphatic capillaries, vessels, and nodes. Drains excess fluid and plasma protein from bodily tissue and returns them to venous system. Prevents edema and produces lymph

Peripheral Vascular System Components

Composed of the Arteries and veins of arms and legs, lymphatic system, and capillaries.

Atelectasis (Collapse)

Condition Collapsed shrunken section of alveoli or an entire lung as a result of (1) airway obstruction (e.g., the bronchus is completely blocked by thick exudate, aspirated foreign body, or tumor); the alveolar air beyond the obstruction is gradually absorbed by the pulmonary capillaries, and the alveolar walls cave in); (2) compression on the lung; and (3) lack of surfactant (hyaline membrane disease). Inspection Cough. Lag on expansion on affected side. Increased respiratory rate and pulse. Possible cyanosis. Palpation Chest expansion decreased on affected side. Tactile fremitus decreased or absent over area. With large collapse, tracheal shift toward affected side. Percussion Dull over area (remainder of thorax sometimes may have hyperresonant note). Auscultation Breath sounds decreased vesicular or absent over area. Voice sounds variable, usually decreased or absent over affected area. Adventitious Sounds None if bronchus is obstructed. Occasional fine crackles if bronchus is patent.

Lobar Pneumonia

Condition Infection in lung parenchyma leaves alveolar membrane edematous and porous; thus red blood cells (RBCs) and white blood cells (WBCs) pass from blood to alveoli. Alveoli progressively fill up (become consolidated) with bacteria, solid cellular debris, fluid, and blood cells, which replace alveolar air. This decreases surface area of the respiratory membrane, causing hypoxemia. History Fever, cough with pleuritic chest pain, blood-tinged sputum, chills, SOB, fatigue. Inspection Increased respirations >24/min. Guarding and lag on expansion on affected side. Children—Sternal retraction, nasal flaring. Palpation Pulse >100 bpm, chest expansion decreased on affected side. Tactile fremitus increased if bronchus patent, decreased if bronchus obstructed. Percussion Dull over lobar pneumonia. Auscultation Tachycardia. Loud bronchial breathing with patent bronchus. Voice sounds have increased clarity; bronchophony, egophony, whispered pectoriloquy present. Children—Diminished breath sounds may occur early. Adventitious Sounds Crackles, fine to medium.

Purpura

Confluent and extensive patch of petechiae and ecchymoses; >3 mm, flat, red to purple, macular hemorrhage. Seen in generalized disorders such as thrombocytopenia and scurvy. Also occurs in old age as blood leaks from capillaries in response to minor trauma and diffuses through dermis.

Talipes Equinovarus (Clubfoot)

Congenital, rigid, and fixed malposition of foot, including (1) inversion, (2) forefoot adduction, and (3) foot pointing downward (equinus). A common birth defect, with an incidence of 1 : 1000 to 3 : 1000 live births. Males are affected twice as frequently as females.

Define the behaviors that are considered in an assessment of a person's mental status.

Consciousness, language, mood and affect, orientation, attention, memory, abstract reasoning, thought process, thought content, and perceptions

Examine the sources that influence the culture and beliefs the student embraces.

Culture; A pattern of shared attitudes, beliefs, self-definitions, norms, roles, and values.Beliefs; An Acceptance that a statement is true or that something exists. a. Family- Family is the main source of where your culture comes from whether it come learning as a kid and continue to participate in those cultures and traditions as you get older b. Friends- Those whom you spend most of your time around tend to rub off on you and open your mind to new cultures, and beliefs and depending on how open/closed minded you are c. Surrounding- Who you associate yourself with, where you hang out, if you go to church, if you don't, what you like to do for fun. etc d. Education- How well educated you are about the different beliefs that are out in this world as well as the hundreds of cultures that are practiced

Incorporate health promotion concepts when performing an assessment of the heart and neck vessels

Cut down on sodium, and fatty food because they leave deposits in the arteries and cause blockages. These blockages can cause

Dacryocystitis (Inflammation of the Lacrimal Sac)

Dacryocystitis is infection and blockage of sac and duct. Pain, warmth, redness, and swelling occur below the inner canthus toward the nose. Tearing is present. Pressure on sac yields purulent discharge from puncta. Dacryoadenitis is an infection of the lacrimal gland (not illustrated). Pain, swelling, and redness occur in the outer third of the upper lid. It occurs with mumps, measles, and infectious mononucleosis or from trauma.

What stage of pressure ulcer would you expect if your apple had a soft dark spot?

Deep Tissue Injury

Fissured or Scrotal Tongue

Deep furrows divide the papillae into small irregular rows. The condition occurs in 5% of the general population and in Down syndrome. The incidence increases with age. (Vertical, or longitudinal, fissures also occur with dehydration because of reduced tongue volume.)

Types of Veins

Deep, superficial, perforator

Secondary Skin Lesions ULCER

Deeper depression extending into dermis, irregular shape; may bleed; leaves scar when heals. Examples: stasis ulcer, pressure injury, chancre.

Scorbutic Gums

Deficiency of vitamin C. Gums are swollen, ulcerated, and bleeding because of vitamin C-induced defects in oral epithelial basement membrane and periodontal collagen fiber synthesis.

Describe the use of assessment, diagnostic reasoning and nursing process in clinical judgment.

Describe the use of diagnostic reasoning in clinical judgement Diag reasoning- analyzing data & drawing conclusion → diag. 1) attend to CUES 2) form HYPOTHESES 3) gather RELATED DATA to hypotheses 4) EVALUATE each hypothesis with new data.Then arrive at final diagnosis. Discuss the use of the nursing process in clinical judgment Nursing process: 1) Assessment 2. Diagnosis 3. Outcome ID 4. Planning 5. Implementation 6. Evalulation Discuss the role of assessment as the starting point of all models of clinical reasoning. -Subjective data + objective data + patient's record, & laboratory studies form the database -Information from database- allows nurse to make a clinical judgment or diagnosis about patient's health state - Key is organization of assessment based on complete factually based data -Evidence based practice importa

Baby Bottle Tooth Decay

Destruction of numerous deciduous teeth may occur in infants and toddlers who take a bottle of milk, juice, or sweetened drink to bed and prolong bottle-feeding past the age of 1 year. Liquid pools around the upper front teeth. Mouth bacteria act on carbohydrates in the liquid, especially sucrose, forming metabolic acids. Acids break down tooth enamel and destroy its protein.

Review breast development in the adolescent female and during pregnancy.

Developmental Considerations: Adolescents At puberty, estrogen stimulates breast changes. Five stages of breast development are correlated with Female Tanner Staging. o Breast buds first sign-Stage 2 Occasionally one breast may grow faster than other, producing a temporary asymmetry. Beginning of breast development precedes menarche by about 2 years. Cultural Competence Timing of puberty is influenced by genetic and environmental factors. Research data indicate age differences in onset of puberty according to different ethnic groups. Menses began at an average age of 12.16 years for black girls and age 13 for white girls. Obesity contributes to the early onset of puberty Developmental Competence: Pregnant Woman Breast changes start during the second month of pregnancy and are an early sign for most women. Hormones stimulate the expansion of the ductal system and supporting fatty tissue and development of the true secretory alveoli. Thus the breasts enlarge and feel more nodular. The nipples grow larger, darker, and more erectile. The areolae become larger and a darker brown as pregnancy progresses, and the tubercles become more prominent. Colostrum- thick yellow fluid is precursor for milk, containing same amount of protein and lactose, but practically no fat. It is rich with antibodies that protect newborn against infection, so breastfeeding is important. o Breasts produce colostrum for first few days after delivery, but it may be expressed as early as the fourth month. Lactation, milk production, begins 1 to 3 days postpartum; whitish color is from emulsified fat and calcium caseinate

What cardiac alterations occurs during pregnancy?

During pregnancy the blood volume increases by 30% to 40%; this creates an increase in stroke volume and cardiac output and an increased pulse rate of 10 to 15 beats per minute. The arterial blood pressure decreases in pregnancy as a result of peripheral vasodilation.

Describe the role cultural heritage and values may play in an individual's nutritional intake.

Each person has unique cultural heritage that may affect nutritional status. Occupation ,class, religion, gender, and health awareness have a great bearing on eating customs. New immigrants may be at nutritional risk because they frequently come with limited food supplies caused by poverty, poor sanitation, war, or political strife. Other factors include: new country with a new language, culture, and society. Faced with unfamiliar foods, food storage, food preparation, and food-buying habits. Many familiar foods are difficult or impossible to obtain. Low income may also limit their access to familiar foods. Borderline deficiencies or adverse nutritional consequences may result. American standard tables of weight, age, height may not work to evaluate immigrants. 24 hour diet recall & 3 day food record inadequate too. Food preferences are interrelated with religious dietary beliefs and practices. Use food for celebrations & rituals.

Paget Disease (Intraductal Carcinoma)

Early lesion has unilateral, clear yellow discharge and dry, scaling crusts, friable at nipple apex. Spreads outward to areola with erythematous halo on areola and crusted, eczematous, retracted nipple. Later lesion shows nipple reddened, ulcerated with bloody discharge, and an erythematous plaque surrounding the nipple. Symptoms include tingling, burning, itching. Except for the expected redness and occasional cracking from initial breastfeeding, any other dermatitis of the nipple area must be explored carefully and referred immediately.

Endogenous Obesity—Cushing Syndrome

Either administration of adrenocorticotropin (ACTH) or excessive production of ACTH by the pituitary stimulates the adrenal cortex to secrete excess cortisol. This causes Cushing syndrome, characterized by weight gain and edema with central trunk and cervical obesity (buffalo hump) and round, plethoric face (moon face). Excessive catabolism causes muscle wasting; weakness; thin arms and legs; reduced height; and thin, fragile skin with purple abdominal striae, bruising, and acne. Note that the obesity here is markedly different from exogenous obesity caused by excessive caloric intake, in which body fat is evenly distributed and muscle strength is intact. (See Chapter 12, Nutrition Assessment, p. 179.)

Angular Cheilitis (Stomatitis, Perlèche)

Erythema, scaling, and shallow and painful fissures at the corners of the mouth occur with excess salivation and Candida infection. It is often seen in edentulous persons and those with poorly fitting dentures, causing folding in of corners of mouth, which creates a warm, moist environment favoring growth of yeast.

Allergic Drug Reaction

Erythematous and symmetric rash, usually generalized. Some drugs produce urticarial rash or vesicles and bullae. History of drug ingestion.

Allens Test

Evaluates patency of the radial/ulnar arteries, implemented when patency is questionable. test that determines the patency of the radial and ulnar arteries by compressing one artery site and observing return of skin color as evidence of patency of the other artery

Exophthalmos (Protruding Eyes)

Exophthalmos is a forward displacement of the eyeballs and widened palpebral fissures. Note "lid lag," in which the upper lid rests well above the limbus and white sclera is visible. Acquired bilateral exophthalmos is associated with thyrotoxicosis.

Polydactyly

Extra digits are a congenital deformity, usually occurring at the 5th finger or the thumb. Surgical removal is considered for cosmetic appearance.

1) Rheumatoid Arthritis is a decrease in skeletal bone mass leading to low bone mineral density (BMD). T OR F

FALSE

Major veins of the Leg

Femoral, Great Saphenous, Popliteal, Small Saphenous

Major arteries of the Leg

Femoral, popliteal, posterior tibial, dorsalis pedis.

Ulnar Deviation or Drift

Fingers drift to the ulnar side because of stretching of the articular capsule and muscle imbalance. Also note subluxation and swelling in the joints and muscle atrophy on the dorsa of the hands. This is caused by chronic RA.

Swan-Neck and Boutonnière Deformity

Flexion contracture resembles curve of a swan's neck, as in metacarpophalangeal joint. Then hyperextension of the PIP joint, and flexion of the DIP joint. It occurs with chronic RA, often accompanied by ulnar drift of the fingers. In boutonnière deformity the knuckle looks as if being pushed through a buttonhole. It is a common deformity.

Eyes Signs Associated With Malnutrition AND Nutrient Deficiency

Foamy plaques (Bitot spots)-Vitamin A Dryness (xerophthalmia) Vitamin A Softening (keratomalacia) Vitamin A Pale conjunctivae Iron, vitamins B6, B12 Red conjunctivae Riboflavin Blepharitis Vitamin B-complex, biotin

Bitot Spots

Foamy plaques of the cornea are the accumulations of keratin that are a sign of vitamin A deficiency. Severe depletion may result in conjunctival xerosis (drying) and progress to corneal ulceration and finally destruction of the eye (keratomalacia).

What stage of pressure ulcer would you expect if you were to bite into the apple and you would get to the core?

Four

Tongue Signs Associated With Malnutrition AND Nutrient Deficiency

Glossitis (beefy red) Vitamin B-complex Pale Iron Papillary atrophy Niacin Papillary hypertrophy Multiple nutrients Magenta or purplish-coloredRiboflavin

Acute Gout

Gout is a painful inflammatory arthritis characterized by excess uric acid in the blood and deposits of urate crystals in the joint space.27 Acute episodes are triggered by surgery, trauma, diuretics, alcohol intake. Episodes are characterized by redness, swelling, heat, and extreme pain such as a continuous throbbing. Increased prevalence in obesity, metabolic syndrome, hypertension, hyperlipidemia

Gingivitis

Gum margins are red and swollen and bleed easily. This case is severe; gingival tissue has desquamated, exposing roots of teeth. Inflammation is usually caused by poor dental hygiene or vitamin C deficiency. The condition may occur in pregnancy and puberty because of changing hormonal balance.

Tophi With Chronic Gout

Hard nodules (tophi) most often in the metatarsophalangeal joint of first toe. Tophi are collections of sodium urate crystals caused by chronic gout in and around the joint. Crystals are stong inflammation triggers that cause extreme painful swelling and joint deformity. They may erode through skin with a chalky discharge.9

List the hemodynamic changes that occur with aging

Hardening of arteries raise blood pressure and aging of heart muscles cause respiratory problems. 1) Pressure/pulse changes a) Isolated systolic HTN: Increase in systolic BP due to thickening and stiffening of the arteries b) Left ventricular wall becomes thicker but the overall size of the heart does not change. c) Pulse pressure increases. d) No change in resting heart rate or cardiac output at rest e) Ability of heart to augment cardiac output with exercise is decreased. 2) Dysrhythmias 3) ECG Changes •Increase in Systolic BP (Isolated systolic HTN) •Thickening and Stiffening of arteries (Vascular Stiffening- Increased Workload) •Increased Collagen and Calcium Deposits •Loss of Elastic Fibers •Left ventricular Wall Thickening •Dysrhythmias (SVT, Ventricular Tach) •Diastolic BP Decreases (higher pulse pressure) •Decreased Ability of Heart to Augment Cardiac Output with Exercise •Ectopic Beats (Extra Beat) •Some ECG Changes •Increase risk of CVD

Describe the procedure for teaching breast self-examination and incorporating health promotion concepts when performing an assessment of the breasts.

Have patient stand in front of mirror. Have patient look and palpate for anything unusual, lightly squeeze nipple assessing for discharge. Raise arms above head while observing breasts for symetrical movement.

Hordeolum (Stye)

Hordeolum is an acute localized staphylococcal infection of the hair follicles at the lid margin. It is painful, red, and swollen—a superficial, elevated pustule at the lid margin. Rubbing the eyes can cause cross-contamination and development of another stye. Managed with warm compresses, topical antibiotic ointment, may be combined with steroid ointment.6

Relate the use of a blood pressure cuff of improper size to the possible findings that may be obtained.

If it is too narrow it yields a falsely high blood pressure

Meth Mouth

Illicit methamphetamine abuse (crystal meth, meth ice) leads to extensive dental caries, gingivitis, tooth cracking, and edentulism. Methamphetamine causes vasoconstriction and decreased saliva, and its use increases the urge to consume sugars and starches and give up oral hygiene. Absence of the buffering saliva leads to increased acidity, and the increased plaque encourages bacterial growth. These conditions and carbohydrate presence produce caries, cracking of enamel, and the damage seen here.

Summarize the spread of the cardiac impulse through the heart

Impulses from the brain activate the SA node to the AV node and on into the bundle branches and bundle fibers. SA node can function by itself without brain activity PQRST EKG

Discuss the influence of culture, religion and spirituality on health and illness perception.

In times of crisis such as serious illness and impending death, religion may be a source of consolation for the person and his or her family. Religious dogma and spiritual leaders may exert considerable influence on the persons decision making concerning acceptable medical and surgical treatment such as vaccinations, choice or healer, and other aspects of illness.- Religious affiliation and practices can support spiritual harmony and health in many ways-Religious affiliation and attendance at religious functions may promote health through social networks and social support systems that buffer and affect stress and isolation- Benefit health by promoting healthy behaviors and lifestyles-Faith benefits health by leading to thoughts of hope, optimism and positive expectation

Spina Bifida

Incomplete closure of posterior part of vertebrae results in a neural tube defect. Seriousness varies from skin defect along the spine to protrusion of the sac containing meninges, spinal fluid, or malformed spinal cord. The most serious type is myelomeningocele (shown here), in which the meninges and neural tissue protrude. In these cases the child is usually paralyzed below the level of the lesion.

Papilledema (Choked Disc)

Increased intracranial pressure causes venous stasis in the globe, showing redness, congestion, and elevation of the disc; blurred margins; hemorrhages; and absent venous pulsations. This is a serious sign of intracranial pressure, usually caused by a space-occupying mass (e.g., a brain tumor or hematoma). Visual acuity is not affected.

Identify age-related changes in the eye.

Infants an Children •Peripheral vision is intact in newborn infant. •Macula, area of keenest vision, is absent at birth but mature by 8 months. •By 3 to 4 months of age, infant establishes binocularity. •Can fixate on a single image with both eyes simultaneously •Consistency changes from that of soft plastic at birth to rigid glass in old age. Aging Adult •Pupil size decreases. • Presbyopia •Lens loses elasticity, becoming hard and glasslike, which decreases ability to change shape to accommodate for near vision. •By age 70, normally transparent fibers of lens begin to thicken and yellow, the beginning of cataracts. •Visual acuity may diminish gradually after age 50, and more so after age 70. •Floaters- accumulated debris; condensed vitreous fibers •Decreased adaptation to darkness •Most common causes of decreased visual functioning in older adults are the following: •Cataract formation •Or lens opacity, resulting from a clumping of proteins in lens •Glaucoma •Or increased intraocular pressure; chronic open-angle glaucoma is most common type •Age-related macular degeneration (AMD) •Or breakdown of cells in macula of retina; loss of central vision •Diabetic retinopathy •Leading cause of blindness in adults ages 25 to 74 years of age •

Conjunctivitis

Infection of the conjunctiva, "pink eye," has red, beefy-looking vessels at periphery but is usually clearer around iris, commonly from viral or bacterial infection, allergy, or chemical irritation. Purulent discharge accompanies bacterial infection. Preauricular lymph node is often swollen and painful, with a history of upper respiratory infection. Symptoms include itching, burning, foreign body sensation, and eyelids stuck together on awakening. Person has normal vision, normal pupil size, and reaction to light.

Achilles Tenosynovitis

Inflammation of a tendon sheath near the ankle (here the Achilles tendon) produces a superficial linear swelling and a localized tenderness along the route of the sheath. Movement of the involved tendon usually causes pain.

Cellulitis

Inflammation of loose, subcutaneous connective tissue. Shows as thickening and induration of auricle with distorted contours.

Describe the use of inspection, palpation, percussion and auscultation as a physical examination technique.

Inspection concentrated watching -always comes first; first thing done in ROS-begins the moment you first meet the patient and develope a general survey Palpation used when you want to assess texture, temperature, moisture, organ location & size, swelling, vibration, pulsation, rigidity/spasticity, presence of lumps/masses and presence of tenderness/pain-follows and confirms points you noted in inspection Percussion -used when you need to map location/size of organs-signals the density of a structure-detects any superficial/abn masses-elicits pain if underlying structure is inflamed-elicits deep tendon reflex w/ percussion hammer Auscultation used when need to hear sounds of the body-done after percussion; except in abdominal region

Inspection, Auscultation, Percussion, Palpation Ovarian Cyst (Large)

Inspection. Curve in lower half of abdomen, toward midline. Everted umbilicus. Auscultation. Normal bowel sounds over upper abdomen where intestines pushed superiorly. Percussion. Top dull over fluid. Intestines pushed superiorly. Large cyst produces fluid wave and shifting dullness. Palpation. Transmits aortic pulsation, whereas ascites does not.

Inspection, Auscultation, Percussion, Palpation for a Tumor

Inspection. Localized distention. Auscultation. Normal bowel sounds. Percussion. Dull over mass if reaches up to skin surface. Palpation. Define borders. Distinguish from enlarged organ or normally palpable structure.

Inspection, Auscultation, Percussion, Palpation Ascites information

Inspection. Single curve. Everted umbilicus. Bulging flanks when supine. Taut, glistening skin due to recent weight gain; increase in abdominal girth. Auscultation. Normal bowel sounds over intestines. Diminished over ascitic fluid. Percussion. Tympany at top where intestines float. Dull over fluid. Produces fluid wave and shifting dullness. Palpation. Taut skin and increased intra-abdominal pressure limit palpation.

Inspection, Auscultation, Percussion, Palpation pregnancy

Inspection. Single curve. Umbilicus protruding. Breasts engorged. Auscultation. Fetal heart tones. Bowel sounds diminished. Percussion. Tympany over intestines. Dull over enlarging uterus. Palpation. Uterine fundus. Fetal parts. Fetal movements.

Inspection, Auscultation, Percussion, Palpation Air or gass

Inspection. Single round curve. Auscultation. Depends on cause of gas (e.g., decreased or absent bowel sounds with ileus); hyperactive with early intestinal obstruction. Percussion. Tympany over large area. Palpation. May have muscle spasm of abdominal wall.

Inspection, Auscultation, Percussion, Palpation obesity

Inspection. Uniformly rounded. Umbilicus sunken (it adheres to peritoneum, layers of fat are superficial to it). Auscultation. Normal bowel sounds. Percussion. Tympany. Scattered dullness over adipose tissue. Palpation. Normal. May be hard to feel through thick abdominal wall.

Pressure Injuries (PI) (Pressure Ulcer, Decubitus Ulcer) Stage 1—Non-Blanchable Erythema

Intact skin is red but unbroken. Localized redness in lightly pigmented skin does not blanch (turn light with fingertip pressure). Dark skin appears darker but does not blanch. May have changes in sensation, temperature, or firmness.

Basal Cell Carcinoma

It is most often on the lower lid and presents as a small, painless nodule with central ulceration and sharp, rolled-out pearly edges. It occurs in older adults; associated with ultraviolet exposure and light skin. It is locally invasive, but metastasis is rare.

Discuss the significance of jugular vein assessment.

Jugular vein assessment help indicate cardiac failure due to reflux of tricuspid valve. the blood can back up

The solid viscera includes which of the following?

KIDNEYS

1) The nurse is assessing an adult American Indian male patient and he reports some abdominal discomfort after drinking milk. The nurse understands that the patient may be describing _________________ intolerance.

LACTOSE

The spleen is located in which quadrant?

LUQ

Scoliosis

Lateral curvature of thoracic and lumbar segments of the spine, usually with some rotation of involved vertebral bodies. Functional scoliosis is flexible; appears with standing and disappears with forward bending. It may compensate for other abnormalities (leg length discrepancy). Structural scoliosis is fixed; the curvature shows both on standing and on bending forward. Note rib hump with forward flexion. When the person is standing, note unequal shoulder elevation, unequal scapulae, obvious curvature, and unequal hip level. Idiopathic scoliosis shows at 10 years of age through adolescence during the peak of the growth spurt; usually not progressive and more common in girls.

Hypopyon

Layer of white blood cells in anterior chamber occurs with iritis and with inflammation in the anterior chamber. Symptons are pain, red eye, and possibly decreased vision.

On which side do cardiac events happen first? SA and AV nodes are wired to which side of the heart first? Which side is more muscular and thus more powerful?

Left

Male Breast Cancer

Less than 1% of breast cancers occur in men.2 It presents as a painless palpable mass—hard, irregular, nontender, fixed to the area; may have nipple retraction. Nipple discharge, is a significant warning of early breast cancer. Note retraction and ulceration shown here. Early spread to axillary lymph nodes occurs because of minimal breast tissue. Because of lack of screening and general awareness, men are diagnosed 5 years later than women and at later stages, with the mean age at 67 years.7a

Internal Jugular Veins

Lie deep to medial to the sternocleidomastoid muscle.

Characteristics of Benign Breast Disease

Likely Age - 30-55 years; decreases after menopause Shape -Round, lobular Consistency -Firm to soft, rubbery Demarcation -Well demarcated Number -Mobile Mobility -Usually multiple; may be single Tenderness -Tender; usually increases before menses; may be noncyclic Skin Retraction -None Pattern of Growth -Size may increase or decrease rapidly; cyclic with menstrual periods Risk to Health - Benign, although general lumpiness may mask other cancerous lump

Characteristics of Cancer

Likely Age - 30-80 years, risk increases after 50 years Shape -Irregular, star-shaped Consistency -Firm to stony hard Demarcation -Poorly defined Number -Single Mobility -Fixed Tenderness -Usually none, can be tender Skin Retraction -Usually Pattern of Growth -Grows constantly Risk to Health - Serious, needs early treatment

Secondary Skin Lesions FISSURE

Linear crack with abrupt edges; extends into dermis; dry or moist. Examples: cheilosis—at corners of mouth caused by excess moisture; athlete's foot.

Primary Contact Dermatitis

Local inflammatory reaction to an irritant in the environment or an allergy. Characteristic location of lesions often gives clue. Often erythema shows first, followed by swelling, wheals (or urticaria), or maculopapular vesicles, scales. Frequently accompanied by intense pruritus. Example here: poison ivy.

Swelling of Menisci

Localized soft swelling from cyst in lateral meniscus shows at the midpoint of the anterolateral joint line. Semiflexion of the knee makes swelling more prominent. Other meniscal injuries present as a sharp acute pain at lateral or medial joint line together with catching, locking, or popping.17 Tears in the menisci occur with severe ligament injury and present with joint instability, swelling, and pain. Suspected tears are referred to orthopedics due to future increased risk of osteoarthritis.

Prepatellar Bursitis

Localized swelling on anterior knee between patella and skin. A tender, fluctuant mass indicates swelling; infection may spread to surrounding soft tissue. The condition is limited to the bursa, and the knee joint itself is not involved. Overlying skin may be red, shiny, atrophic, or coarse and thickened.

Describe the physical changes that may occur because of poorly controlled pain.

Look for nonverbal behaviors of pain, such as guarding, grimacing, moaning, agitation, restlessness, stillness, diaphoresis, or vital sign changes, (increased BP, HR) Keep in mind that individuals react to painful stimuli very differently. tachycardia, elevated BP, increased myocardial oxygen demand, increased cardiac output, hypoventilation, hypoxia, decreased cough, atelectasis, nausea, vomiting, oliguria, urinary retention, spasm, fear, anxiety, impaired cellular immunity, depression, isolation

Pressure Injuries (PI) (Pressure Ulcer, Decubitus Ulcer)Stage 2—Partial-Thickness Skin Loss

Loss of epidermis and exposed dermis. Superficial ulcer looks shallow like an abrasion or open blister with a red-pink wound bed. No visible fat or deeper tissue.

Mild Synovitis

Loss of normal hollows on either side of the patella, which are replaced by mild distention. Occurs with synovial thickening or effusion (excess fluid) as in RA. Also note mild distention of the suprapatellar pouch.

What is a cause of constipation in aging adults? Select All That Apply

MEDS DECREASED PHYSICAL ACTIVITY DIFFICULTY AMBULATING TOILET

Macule vs Patch

Macule Solely a color change, flat and circumscribed, of less than 1 cm. Examples: freckles, flat nevi, hypopigmentation, petechiae, measles, scarlet fever. Patch Macules that are larger than 1 cm. Examples: mongolian spot, vitiligo, café au lait spot, chloasma, measles rash.

Cleft Lip

Maxillofacial clefts are common congenital deformities and occur with strong family history; maternal use of phenytoin (Dilantin), alcohol, and certain drugs; and maternal diabetes. Early treatment preserves the functions of speech and language formation and deglutition (swallowing). A congenital defect, the failure of fusion of the maxillary processes. Wide variation occurs in the extent of cleft formation, from upper lip only, palate only, uvula only, to cleft of the nostril and the hard and soft palates.

Describe the mechanisms that keep blood moving toward the heart in the venous system

Mechanism to keep blood moving by: ØContracting skeletal muscles, pressure gradient caused by breathing and intraluminal valves ØIn the legs, this mechanism is called calf pump or peripheral heart. the contracting skeletal muscles that milk the blood proximally, back toward the heart the pressure gradient caused by breathing, in which inspiration makes the thoracic pressure decrease and the abdominal pressure increase the intraluminal valves, which ensure undirectional flow

Moderate nonproliferative diabetic retinopathy.

Microaneurysms are round, punctate red dots that are localized dilations of a small vessel. Their edges are smooth and discrete. The vessel itself is too small to view with the ophthalmoscope; only the isolated red dots are seen. Dot hemorrhages are deep intraretinal hemorrhages that look splattered on. They are distinguished from microaneurysms by the blurred irregular edges. Lipid (hard) exudates are small yellow-white spots with distinct edges and a smooth, solid-looking surface. They often form a circular or linear pattern. (This is in contrast with drusen, which have a scattered haphazard location [see Fig. 15.34]).

External Jugular Veins

More superficial, lie lateral to the sternocleidomastoid muscle above the clavicle.

Benign ("Fibrocystic") Breast Disease

Multiple tender masses that occur with numerous symptoms and physical findings: (1) swelling and tenderness (cyclic discomfort), (2) nodularity (significant lumpiness, both cyclic and noncyclic), (3) dominant lumps (including cysts and fibroadenomas), (4) nipple discharge (including intraductal papilloma and duct ectasia), and (5) infections and inflammations (including subareolar abscess, lactational mastitis, breast abscess, and Mondor disease). Many women have some form of benign breast disease. Nodularity occurs bilaterally; regular, firm nodules are mobile, well demarcated, and feel rubbery like small water balloons. Pain may be dull, heavy, and cyclic as nodules enlarge. Some women have nodularity but no pain. Cysts are discrete, fluid-filled sacs. Dominant lumps and nipple discharge must be investigated carefully. Nodularity itself is not premalignant but produces difficulty in detecting other cancerous lumps.

List the three types of muscles and their movements.

Muscles are of three types: skeletal, smooth, and cardiac. Skeletal or voluntary muscles are under conscious control Skeletal muscle is composed of bundles of muscle fibers or fasciculi. The skeletal muscle is attached to bone by a tendon—a strong fibrous cord. Skeletal muscles produce the following movements 1. Flexion—Bending a limb at a joint 2. Extension—Straightening a limb at a joint 3. Abduction—Moving a limb away from the midline of the body 4. Adduction—Moving a limb toward the midline of the body 5. Pronation—Turning the forearm so the palm is down 6. Supination—Turning the forearm so the palm is up 7. Circumduction—Moving the arm in a circle around the shoulder 8. Inversion—Moving the sole of the foot inward at the ankle 9. Eversion—Moving the sole of the foot outward at the ankle 10. Rotation—Moving the head around a central axis 11. Protraction—Moving a body part forward and parallel to the ground 12. Retraction—Moving a body part backward and parallel to the ground 13. Elevation—Raising a body part 14. Depression—Lowering a body part

Myocardium

Muscular wall of heart Heart's muscular layer

Identify the effects of some of the age-related changes that take place in the mouth.

Natural tooth loss exacerbated by inadequate dental care, poor oral hygiene, and tobacco use A gradual loss of subcutaneous fat starts during later adult years, making the nose appear more prominent. The nasal hairs grow coarser and stiffer and may not filter the air as well. The hairs protrude and may cause itching and sneezing. The sense of smell may diminish after age 60 years because of a decrease in the number of olfactory nerve fibers. In the oral cavity, the soft tissues atrophy, and the epithelium thins, especially in the cheeks and tongue. This results in loss of taste buds, with about an 80% reduction in taste functioning. Further impairments to taste include a decrease in salivary secretion that is needed to dissolve flavoring agents. Atrophic tissues ulcerate easily, which increases risk for infections such as oral candidiasis. The risk for malignant oral lesions also increases. Many dental changes occur with aging. The tooth surface is abraded. The gums begin to recede, and the teeth begin to erode at the gum line. A smooth V-shaped cavity forms around the neck of the tooth, exposing the nerve and making the tooth hypersensitive. Some tooth loss may occur from bone resorption (osteoporosis), which decreases the inner tooth structure and its outer support. Natural tooth loss is exacerbated by years of inadequate dental care, decay, poor oral hygiene, and tobacco use. If tooth loss occurs, the remaining teeth drift, causing malocclusion. The stress of chewing with maloccluding teeth causes: (1) further tooth loss; (2) muscle imbalance from a mandible and maxilla now out of alignment, which produces muscle spasms, tenderness, and chronic headaches; and (3) stress on the temporomandibular joint, leading to osteoarthritis, pain, and inability to fully open the mouth. A diminished sense of taste and smell decreases the older adult's interest in food and may contribute to malnutrition. Saliva production decreases; saliva acts as a solvent for food flavors and helps move food around the mouth. Decreased saliva flow also occurs with the use of medications that have anticholinergic effects. More than 250 medications have a side effect of dry mouth. The absence of some teeth and trouble with mastication encourage the older person to eat soft foods (usually high in carbohydrates) and decrease meat and fresh vegetable intake. This produces a risk for nutritional deficit for protein, vitamins, and minerals.

Describe the use of critical thinking in diagnostic reasoning and clinical judgment.

Need to sort thru lots of data to judge. 1. ID assumptions 2. ID organized approach → assessment 3. Validation 4. Distinguishing normal vs. abnorm 5. Making hypotheses 6. Clustering related clues 7. Distinguishing relevant vs. irrelev 8. Recognizing inconsistencies 9. ID patterns 10. ID missing info 11. Promote health (risk factors) 12. Diagnosing actual/potential probs

1) A medication given frequently in the ER for chest pain due to cardiac concerns? (5 letter abbreviation)

Nitro

Nodule vs Tumor

Nodule Solid, elevated, hard or soft, larger than 1 cm. May extend deeper into dermis than papule. Examples: xanthoma, fibroma, intradermal nevi. Tumor Larger than a few centimeters in diameter, firm or soft, deeper into dermis; may be benign or malignant, although "tumor" implies "cancer" to most people. Examples: lipoma, hemangioma.

Innocent Murmur

Not associated with any physiologic abnormality, they occur when the ejection of blood into the aorta is turbulent, very common in children and young adults.

Barrel Chest

Note equal AP-to-transverse diameter and that ribs are horizontal instead of the normal downward slope. This is associated with normal aging and also with chronic emphysema and asthma as a result of hyperinflation of lungs.

Central Gray Opacity—Nuclear Cataract

Nuclear cataract shows as an opaque gray surrounded by a black background as it forms in the center of lens nucleus. Through the ophthalmoscope it looks like a black center against the red reflex. It begins after age 40 years and develops slowly, gradually obstructing vision.

Increased Tactile Fremitus

Occurs with conditions that increase the density of lung tissue, thereby making a better conducting medium for vibrations (e.g., compression or consolidation [pneumonia]). There must be a patent bronchus, and consolidation must extend to lung surface for increased fremitus to be apparent.

S4 heart sound

Often termed atrial gallop; can be heard late in diastole, just before S1. Best heard using bell over apical area, rhythm of ta-lub-dub. Extra beat at beginning; usually an abormal finding associated with CAD, HTN, MI. Usually a failing left ventricle.

S3 heart sound

Often termed ventricular gallop; can be heard early in diastole after S2; best heard using bell at apical area, rhythm of lub-dub-ta. Can be nromal in young children and pregnancy. Rarely normal in people over 40. Associated with CHF, myocardial failure, volume overload.

What stage of pressure ulcer would you expect if you have a normal red apple and you're unable to change the red color by touching the apple?

One

Keloid

Overgrowth of scar tissue, which invades original site of trauma. It is more common in darkly pigmented people, although it also occurs in whites. In the ear it is most common at lobule at site of a pierced ear. Overgrowth shown here is unusually large.

Which assessment skill screens for abdominal fluid or masses and is relative to density of abdomen?

PERCUSS

Pressure Injuries (PI) (Pressure Ulcer, Decubitus Ulcer) Stage 3—Full-Thickness Skin Loss

PI extends into subcutaneous tissue and resembles a crater. See subcutaneous fat, granulation tissue, and rolled edges, but not muscle, bone, or tendon.

Pressure Injuries (PI) (Pressure Ulcer, Decubitus Ulcer) Stage 4—Full-Thickness Skin/Tissue Loss

PI involves all skin layers and extends into supporting tissue. Exposes muscle, tendon, or bone, and may show slough (stringy matter attached to wound bed) or eschar (black or brown necrotic tissue), rolled edges, and tunneling.

Chondrodermatitis Nodularis Helicus

Painful nodules develop on rim of helix (where there is no cushioning subcutaneous tissue) as a result of repetitive mechanical pressure or environmental trauma (sunlight). They are small, indurated, dull red, poorly defined, and very painful.

Acute Rheumatoid Arthritis

Painful swelling and stiffness of joints, with fusiform or spindle-shaped swelling of the soft tissue of PIP joints. Fusiform swelling is usually symmetric, the hands are warm, and the veins are engorged. The inflamed joints have a limited range of motion.

Osgood-Schlatter Disease

Painful swelling of the tibial tubercle just below the knee, from overuse injury that places traction and microtrauma on the bone.17 Occurs most in puberty during rapid growth and before closure of the growth plate. Pain increases with kicking, running, bike riding, volleyball, basketball, soccer. It is usually self-limited, and symptoms resolve with rest.

Gingival Hyperplasia

Painless enlargement of the gums, sometimes overreaching the teeth. This occurs with puberty, pregnancy, and leukemia and with long therapeutic use of phenytoin (Dilantin).

Interpret findings obtained during palpation of the abdomen.

Palpation Rebound Tenderness. Assess rebound tenderness when the person reports abdominal pain or when you elicit tenderness during palpation. Choose a site remote from the painful area. Hold your hand 90 degrees, or perpendicular, to the abdomen. Push down slowly and deeply (Fig. 22.27A); then lift up quickly (Fig. 22.27B). This makes structures that are indented by palpation rebound suddenly. A normal, or negative, response is no pain on release of pressure. Perform this test at the end of the examination because it can cause severe pain and muscle rigidity. Pain on release of pressure confirms rebound tenderness, which is a reliable sign of peritoneal inflammation. Peritoneal inflammation accompanies appendicitis. Cough tenderness that is localized to a specific spot also signals peritoneal irritation. Rebound tenderness occurring in the right lower quadrant when pressure is applied to the left lower quadrant (Blumberg sign) may indicate appendicitis. Refer the person with suspected appendicitis for computed tomography (CT) scanning. Inspiratory Arrest (Murphy Sign). Normally, palpating the liver causes no pain. In a person with inflammation of the gallbladder (cholecystitis), pain occurs. Hold your fingers under the liver border. Ask the person to take a deep breath. A normal response is to complete the deep breath without pain. (NOTE: This sign is less accurate in patients older than 60 years; evidence shows that 25% of them do not have any abdominal tenderness).10 When the test is positive, as the descending liver pushes the inflamed gallbladder onto the examining hand, the person feels sharp pain and abruptly stops inspiration midway.

Mammary Duct Ectasia

Pastelike matter in subareolar ducts produces sticky, purulent discharge that may be cream-colored, green, or bloody. A single duct discharge is shown here. Caused by stagnation of cellular debris and secretions in the ducts, leading to obstruction, inflammation, and infection. Itching, burning, or drawing pain occurs around nipple. May have subareolar redness and swelling. Ducts are palpable as rubbery, twisted tubules under areola. May have palpable mass, soft or firm, poorly delineated. Not malignant but needs biopsy.

Interpret findings obtained during percussion of the abdomen.

Percussion Percuss to assess the relative density of abdominal contents and to screen for abnormal fluid or masses. General Tympany First percuss lightly in all four quadrants to determine the prevailing amount of tympany and dullness (Fig. 22.13). Move clockwise. Tympany should predominate because air in the intestines rises to the surface when the person is supine. Dullness occurs over a distended bladder, adipose tissue, fluid, or a mass. Hyperresonance is present with gaseous distention. Liver Span, Splenic Dullness, and Bladder Percussion Traditionally, the upper and lower borders of the liver were identified by percussion to estimate liver span. This technique of measuring liver span underestimates the true liver size because clinicians place the upper border too low and/or the lower border too high.10 Percussion also yields highly variable results between examiners and frequently does not identify hepatomegaly even when present. Therefore, this examination technique is not recommended. Screening for splenomegaly through percussion of splenic dullness is omitted because detection through palpation is more reliable.10 Detection of a distended bladder through percussion is also omitted due to unreliability.10 Bedside bladder scanning with ultrasound is commonly used to estimate bladder volume. The upper liver border is overestimated if chronic obstructive lung disease is present, and both upper and lower edges are obscured if obesity or ascites is present. Costovertebral Angle Tenderness Indirect fist percussion causes the tissues to vibrate instead of producing a sound. To assess the kidney, place one hand over the 12th rib at the costovertebral angle on the back (Fig. 22.14). Thump that hand with the ulnar edge of your other fist. The person normally feels a thud but no pain. (Although this step is explained here with percussion techniques, its usual sequence in a complete examination is with thoracic assessment, when the person is sitting up and you are standing behind.) Sharp pain occurs with inflammation of the kidney or paranephric area, as in pyelonephritis. Palpate Surface and Deep Areas Perform palpation to judge the size, location, and consistency of certain organs and to screen for an abnormal mass or tenderness. Review comfort measures on p. 538. Because most people are naturally inclined to protect the abdomen, you need to use additional measures to enhance complete muscle relaxation. 1. Bend the person's knees. 2. Keep your palpating hand low and parallel to the abdomen. Holding the hand high and pointing down would make anyone tense up. 3. Teach the person to breathe slowly (in through the nose and out through the mouth). 4. Keep your own voice low and soothing. Conversation may relax the person. 5. Try "emotive imagery." For example, you might say, "Now I want you to imagine that you are dozing on the beach, with the sun warming your muscles and the sound of the waves lulling you to sleep. Let yourself relax." 6. With a very ticklish person, keep the person's hand under your own with your fingers curled over his or her fingers. Move both hands around as you palpate; people are not ticklish to themselves. 7. Alternatively perform palpation just after auscultation. Keep the stethoscope in place and curl your fingers around it, palpating as you pretend to auscultate. People do not perceive a stethoscope as a ticklish object. You can slide the stethoscope out when the person is used to being touched. Light and Deep Palpation Begin with light palpation. With the first four fingers close together, depress the skin about 1 cm (Fig. 22.15). Make a gentle rotary motion, sliding the fingers and skin together. Then lift the fingers (do not drag them) and move clockwise to the next location around the abdomen. The objective here is not to search for organs but to form an overall impression of the skin surface and superficial musculature. Save the examination of any identified tender areas until last. This method avoids pain and the resulting muscle rigidity that would obscure deep palpation later in the examination. Muscle guarding. Rigidity. Large masses. Tenderness. As you circle the abdomen, discriminate between voluntary muscle guarding and involuntary rigidity. Voluntary guarding occurs when the person is cold, tense, or ticklish. It is bilateral, and you will feel the muscles relax slightly during exhalation. Use the relaxation measures to try to eliminate this type of guarding, or it will interfere with deep palpation. If the rigidity persists, it is probably involuntary. Involuntary rigidity is a constant, boardlike hardness of the muscles. It is a protective mechanism accompanying acute inflammation of the peritoneum. It may be unilateral, and the same area usually becomes painful when the person increases intra-abdominal pressure by attempting a sit-up. Now perform deep palpation using the technique described earlier but push down about 5 to 8 cm (2 to 3 inches) (Fig. 22.16). Moving clockwise, explore the entire abdomen. To overcome the resistance of a very large or obese abdomen, use a bimanual technique. Place your two hands on top of one another (Fig. 22.17). The top hand does the pushing; the bottom hand is relaxed and can concentrate on the sense of palpation. With either technique note the location, size, consistency, and mobility of any palpable organs and the presence of any abnormal enlargement, tenderness, or masses. Making sense of what you are feeling is more difficult than it looks. Inexperienced examiners complain that the abdomen "all feels the same," as if they are pushing their hand into a soft sofa cushion. It helps to memorize the anatomy and visualize what is under each quadrant as you palpate. Also remember that some structures are normally palpable, as illustrated in Fig. 22.18. Mild tenderness normally is present when palpating the sigmoid colon in the left lower quadrant. Any other tenderness should be investigated. Tenderness occurs with local inflammation, inflammation of the peritoneum or underlying organ, and with an enlarged organ whose capsule is stretched. If you identify a mass, first distinguish it from a normally palpable structure or an enlarged organ. Then note the following: 1. Location 2. Size 3. Shape 4. Consistency (soft, firm, hard) 5. Surface (smooth, nodular) 6. Mobility (including movement with respirations) 7. Pulsatility 8. Tenderness Liver Next palpate for specific organs, beginning with the liver in the RUQ (Fig. 22.19). Place your left hand under the person's back parallel to the 11th and 12th ribs and lift up to support the abdominal contents. Place your right hand on the RUQ, with fingers parallel to the midline. Push deeply down and under the right costal margin. Ask the person to breathe slowly. With every exhalation, move your palpating hand up 1 or 2 cm. It is normal to feel the edge of the liver bump your fingertips as the diaphragm pushes it down during inhalation. It feels like a firm, regular ridge. Often the liver is not palpable and you feel nothing firm. Except with a depressed diaphragm, a liver palpated more than 1 to 2 cm below the right costal margin is enlarged. Record the number of centimeters it descends and note its consistency (hard, nodular) and tenderness (see Table 22.7, Palpation of Enlarged Organs, p. 566). For example, an abnormally firm liver may indicate cirrhosis.10 One variation occurs in people with chronic emphysema, in which the liver is displaced downward by the hyperinflated lungs. Although you palpate the lower edge well below the right costal margin, the overall size is still within normal limits. Hooking Technique. An alternative method of palpating the liver is to stand up at the person's shoulder and swivel your body to the right so that you face the person's feet (Fig. 22.20). Hook your fingers over the costal margin from above. Ask the person to take a deep breath. Try to feel the liver edge bump your fingertips. Scratch Test. This traditional technique uses auscultation to detect the lower border of the liver. Place the stethoscope over the xiphoid process while lightly stroking the skin with one finger up the MCL from the RLQ and parallel to the liver border. When you reach the liver edge, the sound is magnified in the stethoscope. However, there are many variations in the technique, and evidence is mixed as to its value.10 One study found moderate agreement between the results by scratch test and ultrasound.7 The researchers recommend the scratch test if the abdomen is distended, obese, or too tender for palpation or if muscles are rigid or guarded.7 Spleen Normally the spleen is not palpable and must be enlarged 3 times its normal size to be felt. To search for it, reach your left hand over the abdomen and behind the left side at the 11th and 12th ribs (Fig. 22.21A). Lift up for support. Place your right hand obliquely on the LUQ with the fingers pointing toward the left axilla and just inferior to the rib margin. Push your hand deeply down and under the left costal margin and ask the person to take a deep breath. You should feel nothing firm. Imaging by ultrasound is more precise. The spleen enlarges with mononucleosis, trauma, leukemia and lymphomas, portal hypertension, and HIV infection (see Table 22.7). Consider malaria in persons with fever and splenomegaly returning from travel to areas where malaria is endemic. If you feel an enlarged spleen, refer the person but do not continue to palpate it. An enlarged spleen is friable and can rupture easily with overpalpation. Describe the number of centimeters that it extends below the left costal margin. When enlarged, the spleen slides out and bumps your fingertips. It can grow so large that it extends into the lower quadrants. When this condition is suspected, start low so that you will not miss it. An alternative position is to roll the person onto his or her right side to displace the spleen more forward and downward (Fig. 22.21B). Then palpate as described earlier. Kidneys Search for the right kidney by placing your hands together in a "duck-bill" position at the person's right flank (Fig. 22.22A). Press your two hands together firmly (you need deeper palpation than that used with the liver or spleen), and ask the person to take a deep breath. In most people you will feel no change. Occasionally you may feel the lower pole of the right kidney as a round, smooth mass that slides between your fingers. Either condition is normal. The left kidney sits 1 cm higher than the right kidney and is not palpable normally. Search for it by reaching your left hand across the abdomen and behind the left flank for support (Fig. 22.22B). Push your right hand deep into the abdomen and ask the person to breathe deeply. You should feel no change with the inhalation. Aorta Using your opposing thumb and fingers, palpate the aortic pulsation in the upper abdomen slightly to the left of midline (Fig. 22.23). Normally it is 2.5 to 4 cm wide in the adult and pulsates in an anterior direction. Prominent lateral pulsation with aortic aneurysm pushes the examiner's two fingers apart. Palpation may have poor accuracy detecting aneurysm due to interference of the skin and adipose tissue, as well as the retroperitoneal location of the aorta.

Neurologic Signs Associated With Malnutrition AND Nutrient Deficiency

Peripheral neuropathy Thiamine, vitamin B6 Hyporeflexia Thiamine Disorientation or irritability Vitamin B12

Pellagra

Pigmented keratotic scaling lesions resulting from a deficiency of niacin. These lesions are especially prominent in areas exposed to the sun such as hands, forearms, neck, and legs.

German Measles (Rubella)

Pink, papular rash (similar to measles but paler) first appears on face, then spreads. Distinguished from measles by presence of neck lymphadenopathy and absence of Koplik spots.

What type of ulcer is a localized injury to the skin or underlying tissue, usually over a bony prominence?

Pressure

Heart Sounds

Produced by valve closure.

Pleural Friction Fremitus

Produced when inflammation of the parietal or visceral pleura causes a decrease in the normal lubricating fluid. The opposing surfaces make a coarse grating sound when rubbed together during breathing. This sound is best detected by auscultation, but it may be palpable and feels like two pieces of leather grating together. It is synchronous with respiratory excursion. Also called a palpable friction rub.

Dental Caries

Progressive destruction of tooth. Decay initially looks chalky white. Later it turns brown or black and forms a cavity. Early decay shows only on x-ray image. Susceptible sites are tooth surfaces where food debris, bacterial plaque, and saliva collect.

Severe nonproliferative diabetic retinopathy. Note lipid exudates as described and larger flame-shaped hemorrhages that look linear or spindle shaped.

Proliferative diabetic retinopathy (not shown). Neovascularization is new vessel formation that looks like radiating spokes.

Secondary Skin Lesions Lichenification

Prolonged, intense scratching eventually thickens skin and produces tightly packed sets of papules; looks like surface of moss (or lichen).

Ptosis (Drooping Upper Lid)

Ptosis occurs from neuromuscular weakness (e.g., myasthenia gravis with bilateral fatigue as the day progresses), oculomotor cranial nerve III damage, or sympathetic nerve damage (e.g., Horner syndrome) or is congenital as in this example. It is a positional defect that gives the person a sleepy appearance and impairs vision.

1) When auscultating the abdomen the nurse is correct when they begin in which quadrant?

RLQ

During the Abdominal Assessment I will auscultate which quadrant first?

RLQ

Diaper Dermatitis

Red, moist, maculopapular patch with poorly defined borders in diaper area, extending along inguinal and gluteal folds. History of infrequent diaper changes or occlusive coverings. Inflammatory disease caused by skin irritation from ammonia, heat, moisture, occlusive diapers.

Blepharitis (Inflammation of the Eyelids)

Red, scaly, greasy flakes and thickened, crusted lid margins occur with staphylococcal infection or seborrheic dermatitis of the lid edge. Symptoms include burning, itching, tearing, foreign body sensation, and some pain.

Measles

Red-purple maculopapular blotchy rash appears on 3rd or 4th day of illness. Rash appears first behind ears and spreads over face and then over neck, trunk, arms, and legs; looks "coppery" and does not blanch. Also characterized by Koplik spots in mouth—bluish white, red-based elevations of 1 to 3 mm Vaccine refusal has caused a decline in herd immunity and numerous outbreaks of infectious diseases.

Frostbite in ear

Reddish-blue discoloration and swelling of auricle after exposure to extreme cold. Vesicles or bullae may develop, the person feels pain and tenderness, and ear necrosis may ensue.

S3 and S4 heart sound

Referred to as diastolic filling sounds or extra heart sounds; result from ventricular vibration secondary to rapid ventricular filling.

Summarize the mechanics of respiration.

Respiration involves ventilation, diffusion, and perfusion. Ventilation (or breathing) is movement of gases in and out of the lungs; inspiration (or inhalation) is the act of breathing in, and expiration (or exhalation) is the act of breathing out. Unlike heart rate, which is controlled by the autonomic nervous system, ventilation has both autonomic and voluntary control. Diffusion is the exchange of oxygen and carbon dioxide between the alveoli of the lungs and the circulating blood. Perfusion is the exchange of oxygen and carbon dioxide between the circulating blood and tissue cells. Four major functions of respiratory system: 1. Supply O2 to body for energy production 2. Remove CO2 as a waste product of energy reactions 3. Maintain homeostasis of actions- Acid-Base Balance 4. Maintain heat exchange •Four major functions of respiratory system: •Supplying oxygen to the body for energy production •Removing carbon dioxide as a waste product of energy reactions •Maintaining homeostasis (acid-base balance) of arterial blood •By supplying oxygen to blood and eliminating excess carbon dioxide, respiration maintains pH or acid-base balance of blood. •Maintaining heat exchange (less important in humans)

Jugular Venous Pressure

Return blood to the heart from the head to neck by the end of the superior vena cava; helps determine the hemodynamics of the right side of the heart; right-sided heart failure raises pressure and volume; decreased occurs with reduced left ventricular output or reduced blood volume.

Cardiac cycle

Rhythmic flow of blood through heart; Systole (1/3 of cycle); AV valves close signaling beginning of systole (s1); Contraction of ventricular walls; Aortic valve opens/ Blood ejected rapidly; Aortic valves closes signaling end of systole; Diastole; (2/3 of cycle); AV valves are open, ventricles relax; Ventricles fill with blood from atria; Towards end of diastole, atria contract and push lat blood into ventricles (atrial systole)

Post-Polio Muscle Atrophy

Right leg and foot muscle atrophy as a result of childhood polio. Poliomyelitis epidemics peaked in the United States in the 1940s and 1950s. The development of the oral polio vaccine (1962) has almost eradicated the disease. However, thousands of polio survivors have this muscle atrophy.

Cite the risk factors associated with heart disease and stroke

Risk factors include smoking, drinking, poor diet, and high sodium. Nutrition, Diet Alcohol Exercise HTN Serum cholesterol- high levels of LDL (low-density lipoprotein) Physical activity Sex and gender differences •Obesity, Exercise •High blood pressure •Diabetes •Heart and blood vessel diseases •Brain aneurysms or arteriovenous malformations (AVMs) •Infections or conditions that cause inflammation •Age.. •Race and ethnicity •Family history and genetics. •Anxiety, depression, and high stress levels. •Living or working in areas with air pollution. •Other medical conditions •Blood-thinners or other medicines •Other unhealthy lifestyle habits •Overweight and obesity

Which organs are hollow viscera? Select All That Apply.

STOMACH COLON

Intertrigo (Candidiasis)

Scalding red, moist patches with sharply demarcated borders, some loose scales. Usually in genital area extending along inguinal and gluteal folds. Aggravated by urine, feces, heat, and moisture; the Candida fungus infects the superficial skin layers.

Second heart sound (S2)

Second heart sound heard as a result from the closure of the semilunar valves (aortic and pulmonic, usually heard as one sound: dub) Loudest at the Base of the heart.

Left Ventricular Heave

Seen at the apex of the heart.

Right Ventricular Heave

Seen at the sternal border

Secondary Skin Lesions Excoriation

Self-inflicted abrasion; superficial; sometimes crusted; scratches from intense itching. Examples: insect bites, scabies, dermatitis, varicella.

Rickets

Sign of vitamin D and calcium deficiencies in children (disorders of cartilage cell growth, enlargement of epiphyseal growth plates) and adults (osteomalacia).

Chickenpox (Varicella)

Small, tight vesicles first appear on trunk and spread to face, arms, and legs (not palms or soles). Shiny vesicles on an erythematous base are commonly described as the "dewdrop on a rose petal." Vesicles erupt in succeeding crops over several days; they become pustules and then crusts. Intensely pruritic.

Tophi

Small, whitish yellow, hard, nontender nodules in or near helix or antihelix; contain greasy, chalky material of uric acid crystals and are a sign of gout.

Name the components of the thoracic cage and landmarks on the thorax.

Sternum 12 pairs of ribs 12 thoracic vertebrae and diaphragm which forms the floor -Bony structure with conical shape, narrower at apex.-Defined: sternum, 12 pairs of ribs, 12 thoracic vertebrae.-Floor is at diaphragm-Ribs 1-7, attach to strenum by coastal cartilage-Ribs 8-10 attach to coastal cartilage above-Ribs 11-12 "floating" with palpable tips.-Coastochondral junctions: where ribs join cartilage (not papable) -Suprasternal notch - feel this hollow U-shaped depression just above the sternum in-between the clavicles -Sternum - breast bone that has three parts, manubrium, the body, and the xiphoid process -Sternal angle (angle of louis) - articulation of the manubrium and obeyed of the sternum, it is continuous with the second rib -Coastal angle - right and left coastal margins from an angle where they meet at the xiphoid process - Vertebra prominens - start here. flex your head and feel for the most prominent bony spur protruded at the base of the neck -spinous process - count down these knows on the vertebrae which sack together to from the spinal column -Inferior border of the scapula - lower tip is usually at the seventh or eighth rib -Twelfth rib - palpate midway between the spine and the persons side to identify its free tip

Extension

Straightening a body part

Calculating Pulse Deficit

Subtract the radial rate from the apical and record the remainder.

Pallor

Suggests arterial insufficiency. (skin tone)

Anterior Thoracic Landmarks

Suprasternal Notch. Feel this hollow U-shaped depression just above the sternum, between the clavicles. Sternum. The "breastbone" has three parts: the manubrium, the body, and the xiphoid process. Walk your fingers down the manubrium a few centimeters until you feel a distinct bony ridge, the sternal angle. Sternal Angle. Often called the angle of Louis, this is the articulation of the manubrium and body of the sternum, and it is continuous with the 2nd rib. The angle of Louis is a useful place to start counting ribs, which helps localize a respiratory finding horizontally. Identify the angle of Louis, palpate lightly to the 2nd rib, and slide down to the 2nd intercostal space. Each intercostal space is numbered by the rib above it. Continue counting down the ribs in the middle of the hemithorax, not close to the sternum where the costal cartilages lie too close together to count. You can palpate easily down to the 10th rib. The angle of Louis also marks the site of tracheal bifurcation into the right and left main bronchi; it corresponds with the upper border of the atria of the heart, and it lies above the 4th thoracic vertebra on the back. Costal Angle. The right and left costal margins form an angle where they meet at the xiphoid process. Usually 90 degrees or less, this angle increases when the rib cage is chronically overinflated, as in emphysema.

Differentiate between synovial and nonsynovial joints.

Synovial joints are freely movable because bones are separated from one another and enclosed in a join cavity. Cavity is filled with lubricant or synovial fluid. Allow sliding of opposing surfaces, cartilage covers the surface of opposing bones. Nonsynovial joints are united by fibrous tissue or cartilage and are immovable like the sutures in the skull or slightly movable like the vertebrae.

List the contents of the mediastinum, lung borders and five lobes of the lungs.

The Thoracic Cavity: Mediastinum is the middle section of thoracic cavity o Contains esophagus, trachea, heart, & great vessels o Right & Left pleural cavities contain lungs o Lung Borders Anterior: Apex is 3-4cm above inner third of clavicle Base rests on diaphragm o 6 th rib in midclavicular line Lateral: apex of axilla to 7-8 th rib Posterior: C7 apex, T10 base Lobes of the Lung Anterior Reference Lines: Midsternal line Midclavicular line Posterior Reference Lines: Vertebral (midspinal) line Scapular line Lateral Reference Lines: Anterior axillary line Posterior axillary line Midaxillary lines The Thoracic Cavity: Mediastinum is the middle section of thoracic cavity o Contains esophagus, trachea, heart, & great vessels o Right & Left pleural cavities contain lungs o Lung Borders Anterior: Apex is 3-4cm above inner third of clavicle Base rests on diaphragm 6th rib in midclavicular line Lateral: apex of axilla to 7-8th rib Posterior: C7 apex, T10 base Lobes of the Lung: Lobes are separated by fissures ØLungs are paired but not precisely symmetric structures. ØRight lung shorter than left because of underlying liver ØLeft lung narrower than right because heart bulges to left ØRight lung has three lobes, and left lung has two lobes.

Herpes Simplex 1 (HSV-1)

The common cold sores are groups of clear vesicles with a surrounding indurated erythematous base. These evolve into pustules, which rupture, weep, and crust and heal in 4 to 10 days. The most likely site is the lip-skin junction; infection often recurs in the same site. HSV-1 lesion is highly contagious and spread by direct contact. Recurrent infections may be precipitated by sunlight, fever, colds, and allergy.

Conduction Of all organs, the heart has a unique ability—automaticity. The heart can contract by itself, independent of any signals or stimulation from the body. It contracts in response to an electrical current conveyed by a conduction system (Fig. 20.8). Specialized cells in the sinoatrial (SA) node near the superior vena cava initiate an electrical impulse. (Because the SA node has an intrinsic rhythm, it is the "pacemaker.") The current flows in an orderly sequence, first across the atria to the AV node low in the atrial septum. There it is delayed slightly so the atria have time to contract before the ventricles are stimulated. Then the impulse travels to the bundle of His, the right and left bundle branches, and then through the ventricle

The electrical impulse stimulates the heart to do its work, which is to contract. A small amount of electricity spreads to the body surface, where it can be measured and recorded on the electrocardiograph (ECG). The ECG waves are arbitrarily labeled PQRST, which stand for the following elements: P wave—Depolarization of the atria PR interval—From the beginning of the P wave to the beginning of the QRS complex (the time necessary for atrial depolarization plus time for the impulse to travel through the AV node to the ventricles) QRS complex—Depolarization of the ventricles T wave—Repolarization of the ventricles Electrical events slightly precede the mechanical events in the heart.

List the anatomic landmarks of the external nose.

The external nose, shaped like a triangle, consists of the bridge or superior part; the free corner or the tip; the nares, which are the openings at the base of the triangle; and (inside the nares) a vestibule, the columella that divides the two nares and is continuous inside with the nasal septum; and the ala, the lateral outside wing of the nose on each side.

List the significant anatomic features of the heart

The heart wall has numerous layers. It consists of the pericardium, myocardium, and endocardium. The pericardium is a tough, fibrous, double walled sac that surrounds and protects the heart. It has two layers that contain aa few milliliters of serous pericardial fluid. This ensures smooth, friction free movement of the heart muscle. The pericardium is adherent to the great vessels, esophagus, sternum, and pleurae and is anchored to the diaphragm. The myocardium is the muscular wall of the heart, it does the pumping. The endocardium is the thin layer of endothelial tissue that lines the inner surface of the heart chambers and valves. There are four chambers in the heart and they are separated by swinging door like structures, called valves, whose main purposes is to prevent backflow of blood. There are four valves in the heart, there are two atrioventricular valves that separate the atria and the ventricles. The right av valve is the tricuspid, and the left av valve is the bicuspid or mitral valve. Right Atrium, Right Ventricle, Tricuspid Valve, Pulmonary Valve, Left Atuim, Left Ventricle, Mitral Valve, Aortic Valve, Septum

Ectropion

The lower lid is loose and rolling out (eversion), does not approximate to eyeball. Puncta cannot siphon tears effectively; thus excess tearing results. The eyes feel dry and itchy because the tears do not drain correctly. Exposed palpebral conjunctiva increases risk for inflammation. It occurs in aging from atrophy of elastic and fibrous tissues but may result from trauma, chronic inflammation, or Bell palsy.

Entropion

The lower lid rolls in (inversion) because of spasm of lids or scar tissue contracting. Constant rubbing of lashes may irritate cornea, leading to tearing and red eye. The person feels a "foreign body" sensation.

Identify the structures and function of the oral cavity.

The mouth is the first segment of the digestive system and an airway for the respiratory system. The oral cavity is a short passage bordered by the lips, palate, cheeks, and tongue. It contains the teeth and gums, tongue, and salivary glands

Herniated Intervertebral Disc

The nucleus pulposus (the center of the disc) ruptures into the spinal canal and puts pressure on the local spinal nerve root, causing pain and inflammation. Usually occurs from strenuous activities (lifting, twisting, continuous flexion with lifting, fall on buttocks), mostly in men 20 to 45 years of age, more in smokers.6 Lumbar herniations occur mainly in interspaces L4 to L5 and L5 to S1. NOTE: Sciatic pain, numbness, and paresthesia of involved dermatome; listing away from affected side; decreased mobility; low back tenderness; and decreased motor and sensory function in leg. Straight leg raising tests reproduce sciatic pain

Semilunar Valves are the what

The pulmonary valve and aortic valve.

Secondary Skin Lesions ATROPHIC SCAR

The resulting skin level is depressed with loss of tissue; a thinning of the epidermis. Example: striae.

Nipple Retraction.

The retracted nipple looks flatter and broader, like an underlying crater. A recent retraction suggests cancer, which causes fibrosis of the whole duct system and pulls in the nipple. It also may occur with benign lesions such as ectasia of the ducts. Do not confuse retraction with the normal long-standing type of nipple inversion, which has no broadening and is not fixed.

Dimpling in the breast

The shallow dimple (also called a skin tether) shown here is a sign of skin retraction. Cancer causes fibrosis, which contracts the suspensory ligaments. The dimple may be apparent at rest, with compression, or with lifting of the arms. Also note the distortion of the areola here as the fibrosis pulls the nipple toward it.

Smooth, Glossy Tongue (Atrophic Glossitis)

The surface is slick and shiny; the mucosa thins and looks red from decreased papillae. Accompanied by dryness of tongue and burning. Occurs with vitamin B12 deficiency (pernicious anemia), folic acid deficiency, and iron deficiency anemia. Here also note angular cheilitis.

Crust Secondary Skin Lesions

The thickened, dried-out exudate left when vesicles/pustules burst or dry up. Color can be red-brown, honey, or yellow, depending on fluid ingredients (blood, serum, pus). Examples: impetigo (dry, honey-colored), weeping eczematous dermatitis, scab after abrasion.

Enlarged Tongue (Macroglossia)

The tongue is enlarged and may protrude from the mouth. The condition is not painful but may impair speech development. Here it occurs with Down syndrome; it also occurs with cretinism, myxedema, and acromegaly. A transient swelling also occurs with local infections.

Atrioventricular Valve (AV) are what

The tricuspid valve and mitral (bicuspid) valve. T

Identify the external anatomic features of the eye.

The upper eyelid (note: covers half of the iris), palpebral fissure, pupil, iris, sclera, medial and lateral canthus, caruncle, the limbus, and lower eyelid (note: covers just at the limbus).

Bifid Uvula

The uvula looks partly severed and may indicate a submucous cleft palate, which feels like a notch at the junction of the hard and soft palates. This may affect speech development because it prevents necessary air trapping. The incidence is more common in American Indians.

Describe the shape and surface landmarks of the spine.

The vertebrae are 33 connecting bones stacked in a vertical column. You can feel their spinous processes in a furrow down the midline of the back. The furrow has paravertebral muscles mounded on either side down to the sacrum, where it flattens. Humans have 7 cervical, 12 thoracic, 5 lumbar, 5 sacral, and 3 or 4 coccygeal vertebrae. The following surface landmarks will orient you to their levels: • The spinous processes of C7 and T1 are prominent at the base of the neck. • The inferior angle of the scapula normally is at the level of the interspace between T7 and T8. • An imaginary line connecting the highest point on each iliac crest crosses L4. • An imaginary line joining the two symmetric dimples that overlie the posterior superior iliac spines crosses the sacrum.

Name the paranasal sinuses and their functions.

There are four pairs of sinuses, two of which are accessible to examination—the frontal and maxillary sinuses. The ethmoid and sphenoid sinuses are not accessible to examination. The sinuses lighten the weight of the skull bones, serve as resonators for sound production, and provide mucus. The paranasal sinuses are air-filled pockets within the cranium (Fig. 17.3). They communicate with the nasal cavity and are lined with the same type of ciliated mucous membrane. They lighten the weight of the skull bones; serve as resonators for sound production; and provide mucus, which drains into the nasal cavity. The sinus openings are narrow and easily occluded, which may cause inflammation or sinusitis.

Iritis (Circumcorneal Redness)

There is a deep, dull red halo around the iris and cornea. Note that redness is around the iris, in contrast with conjunctivitis, in which redness is more prominent at the periphery. Pupil shape may be irregular from swelling of iris. Person also has marked photophobia, constricted pupil, blurred vision, and throbbing pain. Warrants immediate referral.

Marfan Syndrome

This inherited connective tissue disorder is characterized by tall, thin stature (≥95th percentile), arachnodactyly (long, thin fingers), hyperextensible joints, arm span greater than height, pubis-to-sole measurement exceeding crown-to-pubis measurement, sternal deformity (note pectus excavatum), high-arched narrow palate, narrow face, and pes planus (flat feet). Early morbidity and mortality occur as a result of cardiovascular complications such as mitral regurgitation and aortic dissection.

Narrowed (Attenuated) Arteries

This is a generalized decrease in arteriole diameter. The light reflex also narrows. It occurs with severe hypertension (shown above on the right) and with occlusion of the central retinal artery and retinitis pigmentosa.

Plugged Duct

This is common when milk is not removed completely because of poor latching, ineffective suckling, infrequent nursing, or switching to second breast too soon. There is a tender lump that may be reddened and warm to touch. No infection. It is important to keep breast as empty as possible and milk flowing. The woman should nurse her baby frequently on affected side first to ensure complete emptying and manually express any remaining milk. A plugged duct usually resolves in less than 1 day.

Anorexia Nervosa

This serious mental health disorder is characterized by severe and life-threatening weight loss in an otherwise healthy person. Behavior is characterized by fanatic concern about weight, aversion to food, distorted body image (perceives self as fat despite skeletal appearance), starvation diets, frenetic exercise patterns, and striving for perfection. Results in amenorrhea in females.

Petechiae

Tiny punctate hemorrhages, 1 to 3 mm, round and discrete; dark red, purple, or brown in color. Caused by bleeding from superficial capillaries; will not blanch. May indicate abnormal clotting factors. In dark-skinned people petechiae are best visualized in the areas of lighter melanization (e.g., the abdomen, buttocks, and volar surface of the forearm). When the skin is black or very dark brown, petechiae cannot be seen in the skin. Most of the diseases that cause bleeding and microembolism formation such as thrombocytopenia, subacute bacterial endocarditis, and other septicemias are characterized by petechiae in the mucous membranes and on the skin. Thus you should inspect for petechiae in the mouth, particularly the buccal mucosa, and in the conjunctivae.

List the structures that compose the respiratory dead space.

Trachea and Bronchial Tree:

Discuss the location and functions of the trachea and bronchial tree.

Trachea is anterior to esophagus. Begins at cricoid cartilage and bifurcates below sternal angle into right and left bronchi o Right main bronchis is shorter, wider and more vertical than left main bronchus Function to transport gases to and from lung parenchyma They constitute dead space (of about 150 ml), no gas exchange occurs Acinus is functional respiratory unit o Consists of bronchioles, alveolar ducts, alveolar sacs, alveoli

Battle Sign

Trauma to the side of the head may lead to a basilar skull fracture involving the temporal bone. This shows as ecchymotic discoloration just posterior to the pinna and over the mastoid process. A look inside the ear canal may show hemotympanum as well

1) The _______________________ valve is between the right atrium and right ventricle.

Tricuspid

The opening of valves is silent. True or False?

True

Pustule

Turbid fluid (pus) in the cavity. Circumscribed and elevated. Examples: impetigo, acne.

What should be done every two hours if a patient is immobile?

Turn

Inversion

Turning the sole of the foot inward

State why two distinct components to each heart sound exist

Two separate sets of valves closing on two various locations create the two distinct heart sounds. In the first heart sound the mitral component (M1) closes just before the tricuspid component (T1). And with S2, aortic closure (A2) occurs slightly before pulmonic closure (P2). I. First heart sound (S1) (lub) A. Occurs with closure of AV valves—signals beginning of systole B. Tricuspid and Mitral Close 1. Pulmonic and Aortic Open II. Second heart sound (S2) (dub) A. Occurs with closure of semilunar valves—signals end of systole B. Pulmonic and Aortic Close 1. Tricuspid and Mitral Open

What stage of pressure ulcer would you expect if you had an apple completely covered with caramel so you really don't know the state of the apple underneath?

Unstageable

Peritonsillar Abscess

Untreated acute streptococcal pharyngitis may cause suppurative complications, peritonsillar abscess, or suppurative thrombophlebitis. Thrombophlebitis is Lemierre syndrome, a rare but life-threatening condition caused by the gram-negative F. necrophorum, leading to sepsis. The two major red flags are worsening symptoms or neck swelling, along with fever and decreased range of motion.16

List the pulses accessible to examination.

VEINS Accessible to examination: ØJugular veins ØVeins in the leg: •Deep—femoral and popliteal •Superficial—great and small saphenous •Perforators—connecting veins that join two sets ARTERIES Accessible to examination: ØTemporal and carotid arteries ØArteries in the arm—brachial bifurcates into ulnar and radial ØArteries in the leg—femoral, popliteal, anterior tibial to dorsalis pedis and posterior tibial to plantar arteries Peripheral arterial disease (PAD) affects noncoronary vessels and refers to arteries affecting the limbs. carotid pulse brachial pulse temporal pulse femoral pulse popliteal pulse dorsalis pedis pulse posterior tibial pulse

Blood Flow Mechanisms

Valves, muscular contraction, pressure gradient; 3 systems. Failure of mechanisms can result in impeded venous return, venous stasis.

Raynoud's Disease

Vascular disorder caused by vasoconstriction or vasospasm of fingers or toes. Cold fingers/toes are common signs of this.

Plantar Wart

Vascular papillomatous growth is caused by human papillomavirus and occurs on the sole of the foot, commonly at the ball and has small dark spots. Although it looks like a callus, it is extremely painful. The wart is tender if you pinch it side to side, whereas a callus is tender to direct pressure.

Vesicle vs Bulla

Vesicle Elevated cavity containing free fluid, up to 1 cm; a "blister." Clear serum flows if wall is ruptured. Examples: herpes simplex, early varicella (chickenpox), herpes zoster (shingles), contact dermatitis. Bulla Larger than 1 cm diameter; usually single chambered (unilocular); superficial in epidermis; thin-walled and ruptures easily. Examples: friction blister, pemphigus, burns, contact dermatitis.

Coronary Arteries

Vessels that lie over the heart. When things go wrong with these, you have a myocardial infarction (heart attack)

Rhonchal Fremitus

Vibration felt when inhaled air passes through thick secretions in the larger bronchi. This may decrease somewhat by coughing.

Pulse Amplitude: 1+

Weak (Pulse amp)

Syndactyly

Webbed fingers are a congenital deformity requiring surgical separation. The metacarpals and phalanges of the webbed fingers are different lengths and the joints do not line up. To leave the fingers fused would thus limit their flexion and extension.

Wheal vs Urticaria (hives)

Wheal Superficial, raised, transient, and erythematous; slightly irregular shape from edema (fluid held diffusely in the tissues). Examples: mosquito bite, allergic reaction, dermographism. Urticaria Wheals coalesce to form extensive reaction, intensely pruritic.

Developmental Dysplasia of the Hip

With a dislocated hip the head of the femur is displaced out of the cup-shaped acetabulum. The degree varies; subluxation may occur as stretched ligaments allow partial displacement of femoral head, and acetabular dysplasia may develop because of excessive laxity of hip joint capsule. Occurrence is 1 : 500 to 1 : 1000 births; common in girls by 7 : 1 ratio. Signs include limited abduction of flexed thigh (illustration A), positive indications of Ortolani sign, asymmetric skin creases or gluteal folds, unequal knee elevation (illustration B), limb length discrepancy, and positive indication of Trendelenburg sign in older children.

Excessive Cup-Disc Ratio

With primary open-angle glaucoma, the increased intraocular pressure decreases blood supply to retinal structures. The physiologic cup enlarges to more than half of the disc diameter, vessels appear to plunge over edge of cup, and vessels are displaced nasally. This is asymptomatic, although the person may have decreased vision or visual field defects in the late stages of glaucoma.

Ankylosis

Wrist in extreme flexion with ruptures of wrist and finger extensors, caused by severe rheumatoid arthritis (RA). This is a functionally compromised hand because, when the wrist is palmar flexed, a good deal of power is lost from the fingers, and often the thumb cannot oppose the fingers.

(((((((((((((((((())))))))))))))))))) Incorporate health promotion concepts when performing an assessment of the thorax and lungs.

`as part of assessment include health promotion. example exposure to secondhand smoke (or environmental tobacco smoke) which increase pt risk of adverse health effect

Lesion LINEAR,

a scratch, streak, line, or stripe.

1) Calculate the Cardiac Output (L/min) HR= 90 bpm and SV = 70 mL

a. 6.3 L/min

1) During shift report the nurse reports a patient is having dysphagia. The nurse receiving report would:

a. Ask the patient about difficulty swallowing

1) A nurse is reviewing a chart and sees the medical term eructation. The nurse understands the meaning of this word as:

a. Belching

1) A 15-year-old softball player is having difficulty pitching the ball due to shoulder pain. Which range of motion (ROM) may be affected?

a. Circumduction

1) A functional assessment for ADLs includes:

a. Climb upstairs b. Walking downstairs c. Picking up object from floor e. Rise up from lying in bed

1) The nurse is performing an assessment on an adult. The adult's vital signs are normal, and capillary refill time is 5 seconds. What should the nurse do next?

a. Consider this a delayed capillary refill time, and investigate further.

1) The nurse hears no sounds from the RUQ abdomen after listening for 1 minute, what should the nurse do next?

a. Continue to listen for 5 minutes

1) The best prevention for Osteoporosis is which of the following?

a. Fast walking

1) A patient complains of pain 6/10 in the right upper quadrant, the patient reports eating a large meal that had very fatty content. Which organ is in the RUQ that may be causing this pain?

a. Gallbladder

1) The nurse is preparing to assess the dorsalis pedis artery. Where is the correct location for palpation?

a. Lateral to the exterior tendon of the great toe

1) The nurse is reviewing an assessment of a patient's peripheral pulses and notices that the documentation states that the radial pulses are "2+." The nurse recognizes that this reading indicates what type of pulse?

a. Normal

1) The nurse is performing a well-child checkup on a 5-year-old boy. The child has no current condition that would lead the nurse to suspect an illness. His health history is unremarkable, and he received immunizations 1 week ago. Which of these findings should be considered normal in this patient?

a. Palpable firm, small, shotty, mobile, and nontender lymph nodes

1) The nurse is examining the lymphatic system of a healthy 3-year-old child. Which finding should the nurse expect?

a. Palpable superficial lymph nodes

1) A 65-year-old patient is experiencing pain in his left calf when he exercises which disappears after resting for a few minutes. What problem in the left leg does this indicate?

a. Partial blockage of an artery

1) During an assessment of an older adult, the nurse should expect to which finding as a normal physiologic change associated with the aging process?

a. Peripheral blood vessels growing more rigid with age, producing a rise in systolic blood pressure

1) The nurse is reviewing the risk factors for venous disease. Which of these situations best describes a person at highest risk for the development of venous disease?

a. Person who has been on bed rest for 4 days

1) During an assessment, a patient tells the nurse that her fingers often change color when she goes out in cold weather. She describes these episodes as her fingers first turning white, then blue, then red with a burning, throbbing pain. What does the nurse suspect?

a. Raynaud phenomenon

1) A patient is having difficulty turning their head to the right. This is an impairment in which range of motion (ROM)

a. Rotation

1) The nurse is assessing a patient and notices contracture of the right hand. The nurse knows this term means:

a. Shortening of a muscle leading to limited range of motion of a joint

1) A patient is recovering from several hours of orthopedic surgery. During an assessment of the patient's lower legs, the nurse will monitor for signs of acute venous symptoms. Signs of acute venous symptoms include which of the following? (Select all that apply.)

a. Warm, red, swollen calf b. Sudden onset c. Intense, sharp pain, with the deep muscle tender to the touch

1) The divisions of the spinal vertebrae include

a. cervical, thoracic, lumbar, sacral, and coccygeal.

1) Crepitation is an audible sound that is produced by

a. roughened articular surfaces moving over each other.

Ascites

abnormal accumulation of serous fluid within the peritoneal cavity, associated with heart failure, cirrhosis, cancer or portal HTN

Lordosis

abnormal anterior curvature of the lumbar spine (sway-back condition)

Splenomegaly

abnormal enlargement of spleen

Hernia

abnormal protrusion of bowel through weakening in abdominal musculature

Costovertebral Angle

angle formed by the 12th rib and the vertebral column on the posterior thorax, overlying the kidney

Lesion POLYCYCLIC,

annular lesions grow together (e.g., lichen planus, psoriasis).

All People Enjoy Time Magazine

aortic, pulmonic, erb's point, tricuspid, mitral

Cherry (senile) angiomas

are small (1 to 5 mm), smooth, slightly raised bright red dots that commonly appear on the trunk in all adults older than 30 years (Fig. 13.8). They normally increase in size and number with aging and are not significant.

Precordium

area of the chest wall overlying the heart and great vessles

Relate the structure and functions of arteries and veins.

artery function- to supply oxygen and essential nutrients to the tissues artery structure- contain elastic fibers, which allows their walls to stretch with systole and recoil with diastole. contain muscle fibers, which control the amount of blood delivered to the tissues. Each heartbeat creates a pressure wave, which makes the arteries expand and then recoil vein function- absorb CO2 and waste products from the periphery and carry them back to the heart. vein structure- lie close to the surface of the skin, course of the vein parallel to the arteries

Types of adventitious breath sounds

atelectatic, absence of sounds, crackles (rales), rhonchi, wheeze, consolidation, stridor, pleural friction rub

Axillary. Temp

axillary route is safer and more accessible than the rectal route; 96.6-98.6 97.6

Describe the data or information that must be gathered for each category of a health history.

biographic data- name, birthdate, gender, race; record who gives this info.reason for seeking care- Sxs/signs, record whatever the pt says, include timeframe. present health or Hx of present illness- PQRSTU mnemonic. past Hx- childhood illness, accidents or injuries, chronic illnesses, hospitalizations, operations, obstetric HX, immunizations, allergies, current meds, last exam date. family Hx- health of close family members: heart disease, high BP, mental illness, seizures, cancer, etc. review of systems: absence or presence of all Sxs. functional assessment (ADLs)- self concept/self esteem, occupational health, activity and exercise, sleep and rest, nutrition and elimination, interpersonal relationships and resources, coping and stress management, environment and work hazards, spiritual resources, personal habits, illicit drugs, intimate partner violence

Bruit

blowing, swooshing sound heard through a stethoscope when an artery is partially occluded A blowing or swishing sound caused by turbulent blood through a narrowed vessel; indicative of occlusive artery disease. (Hear)

Base of Heart

border area of heart's outline located at the 3rd right and left intercostal spaces Upper portion of heart

Clubbing

bulbous enlargement of distal phalanges of fingers and toes that occurs with chronic cyanotic heart and lung conditions

Cite the location of superficial groups of lymph nodes that are accessible to examination.

cervical nodes- drain the head and neck axillary nodes- drain the breast and upper arm epitrochlear node- the antecubital fossa ad drains the hand and lower arm inguinal node- in the groin drain most of the lymph of the lower extremity, the external genitalia, and the anterior abdominal wall

Lesion GROUPED,

clusters of lesions (e.g., vesicles of contact dermatitis).

Carotid Pulse

coincides with ventricular systole. Carotid pulse and apical pulse should be in sync

An acquired condition is vitiligo, the

complete absence of melanin pigment in patchy areas of white or light skin on the face, neck, hands, feet, and body folds and around orifices (Fig. 13.3, B). Vitiligo occurs in all people, although dark-skinned people are more severely affected and potentially suffer a greater threat to their body image.

Describe the significant features of the head.

cranial bones, sutures and facial bones-articulate at sutures and are immovable

Ischemia

deficiency of arterial blood to a body part due to constriction or obstruction of a blood vessel

Umbulicus

depression on the abdomen marking site of entry of umbilical cord

Varicose Veins

dilated tortuous veins with incompetent valves

Cyanosis

dusky blue mottling of the skin and mucous membranes due to excessive amount of reduced hemoglobin in the blood Suggests venous insufficiency when dependent. (skin tone)

Precardial fluid

ensures smooth, friction free movement of the heart muscle

Kyphosis

excessive outward curvature of the spine, causing hunching of the back.

Cecum

first or proximal part of the large intestine

Tympany

high-pitched, musical, drum-like percussion note heard when percussing over the stomach and intestine

Midclavicular line (MCL)

imaginary vertical line bisecting the middle of the clavicle in each hemithorax

Pitting Edema

indentation left after examiner depresses the skin over swollen edematous tissue

Describe the unique nutritional needs for various developmental periods throughout the life cycle.

infants/children- increase length and brain size. Breastfeed recommended. No cow's milk, no low-fat/skim. Need fatty acids. adolescence- rapid growth & endocrine & hormonal changes. Calorie/protein increase for bone and growth and muscle development. Calcium & Iron. 3 meals + snacks pregnancy- for synthesis of maternal and fetal tissues: increase calories, protein, vitamins, minerals (iron, folate, zinc). adult- growth and nutrient needs stabilize. lifestyle factors may lead to problems of hypertension, obesity, atherosclerosis, cancer, diabetes. Prevent/delay onset of disease. aging adult- increased risk for undernutrition or over nutrition. protein, vitamins, minerals (vit D & calcium), nutrient-dense food.

Relate the anatomic developmental differences that alter hearing.

infants: euastation tube is short, wide and more horizontal, this allows easier pathogen access nasophyranx aging: coarse and stiff cilia lining the ear, cerium impacted, nerve degeneration in inner ear ´Presbycusis: type of hearing loss that occurs with aging, even in people living in quiet environment

Thrombophlebitis

inflammation of the vein associated with thrombus formation

Viscera

internal organs

Milia

is a common variation (Fig. 13.21); you will note tiny white papules on the forehead and eyelids, also on cheeks, nose, and chin, caused by sebum that occludes the opening of the follicles. Tell parents not to squeeze the lesions; milia resolve spontaneously within a few weeks.

The café au lait spot

is a large round or oval patch of light brown pigmentation (thus the name coffee with milk), which is usually present at birth ( Six or more café au lait macules, each more than 1.5 cm in diameter, are diagnostic of neurofibromatosis, an inherited neurocutaneous disease.

e mongolian spot

it is a blue-black-to-purple macular area at the sacrum or buttocks but sometimes on the abdomen, thighs, shoulders, or arms. It is caused by deep dermal melanocytes. It gradually fades during the first year. By adulthood these spots are lighter but are frequently still visible.

1) The largest joint in the body is the:

knee

Mitral valve aka the bicuspid valve.

left atrioventricular valve separating the left atrium and ventricle 5th interspace at around midclavicular line

Lesion CONFLUENT,

lesions run together (e.g., urticaria [hives]).

Lesion ZOSTERIFORM,

linear arrangement along a unilateral nerve route (e.g., herpes zoster).

Borborygmi

loud, gurgling bowel sounds signaling increased motility or hyperperistalsis; occurs with early bowel obstruction, gastroenteritis, diarrhea

Costal Margin

lower border of rib margin formed by the medial edges of the 8th, 9th, and 10th ribs

xiphoid process

lower, narrow portion of the sternum

Depression

lowering a body part

Abduction

movement away from the midline

Pronation

movement that turns the palm down

Supination

movement that turns the palm up

Adduction

movement toward the midline

Retraction

moving a body part backward

Protraction

moving a body part forward

Epigastrium

name of abdominal region between the costal margins

Suprapubic

name of abdominal region just superior to pubic bone

Describe the differentiation between normal and abnormal skin color for various ethnic groups.

normal abnormal light-skin: pinkish tan to ruddy pallor (pale)dark tan cyanosis (dusky blue) dark-skin: light to dark brown pallor (yellow-brown, dull)ashen gray, dull cyanosis (dark dull)erythema (purplish)jaundice (sclera, yellow near limbus)

Physiologic splitting

normal variation on S2 heard as two separate components during inspiration

First heart sound (s1)

occurs with closure of the atrioventricular valves signaling the beginning of systole. Heard best at the apex First heart sound heard from the closure of the AV Valves (the mitral and tricuspid, usually heard as one sound: lub) Loudest at Apex of the heart

Relate developmental care during a health history for a child or older adult.

older adult functional assessment: (ADLs) self concept/self esteem, occupation, activity and exercise, sleep and rest, nutrition and elimination, interpersonal relationships and resources, coping and stress management, environment and home safety hazards

Ulcer

open skin lesion extending into dermis, with sloping of necrotic inflammatory tissue

Lesion TARGET,

or iris, resembles iris of eye, concentric rings of color in lesions (e.g., erythema multiforme).

Visceral Pain

originating from the larger internal organs, often described as dull, deep, squeezing, or cramping

State the significance of skin tone changes

pallor (pale) = oxygenate hemoglobin in blood is decreased erythema (intense redness) = excess blood in dilated superficial capillaries cyanosis (bluish) = tissues have high levels of deoxygenated blood jaundice (yellow) = increased amounts of bilirubin in blood

Identify the structures and landmarks of the neck.

passage for many structures, contains blood vessels: carotid arteries and jugular veins; sternocleidomastoid muscle is responsible for head rotation and flexion; trapezius muscle moves the shoulders and extends and turns the head; thyroid gland straddles the trachea in the middle of the neck; cricoid cartilage; thyroid cartilage "adams apple"

List the risk factors for venous stasis.

people who undergo prolonged standing sitting or bed rest because they do not benefit from the milking action that walking accomplishes hypercoagulable states and vein wall trauma are other factors that increase risk for venous disease

Pulse

pressure wave created by each heartbeat, palpable at body sites where the artery lies close to the skin and over a bone

Pulmonic regurgitation

pulmonic insufficiency; backflow of blood through incompetent pulmonic valve into right ventricle

Atrium

reservoir for holding blood; thinned walled reservoir

Tricuspid valve

right atrioventricular valve separating the right atrium and ventricle; Left lower sternal border

Pulmonic Valve aka Pulmonary Semilunar Valve

right semilunar valve separating the right ventricle and pulmonary artery Location: 2nd left interspace

Angle of Louis (sternal angle)

site of the second sternocostal joint manubriosternal angle, the articulation of the manubrium and body of the sternum, continuous with the second rib

Lymph Nodes

small oval clumps of lymphatic tissue located at grouped intervals along lymphatic vessels

Name the related organs and functions of the lymphatic system.

spleen: 1) to destroy old red blood cells 2)to produce antibodies 3) to store red blood cells 4) to filter microorganisms from the blood Tonsils: (palatine, pharyngeal, and lingual) respond to local inflammation Thymus: important in developing the T lymphocytes of the immune system in children. The B lymphocytes originate in the bone marrow and mature in the lymphoid tissue. Functions of lymphatic system are to Øconserve fluid and plasma proteins that leak out of capillaries. Øform a major part of immune system that defends body against disease. Øabsorb lipids from intestinal tract.

Which organs are hollow viscera

stomach, gallbladder, small intestine, colon, bladder

Lymphedema

swelling of extremity due to obstructed lymph channel, nonpitting results from blocked lymphatic circulation, which may be caused by breast surgery. Usuall affects one extremity, causing induration and nonpitting edema.

Edema

swelling of legs or dependent body part due to increased interstitial fluid

Syncope

temporary loss of consciousness due to decreased cerebral blood flow (fainting); caused by ventricular asystole, bradycardia, or ventricular fibrillation

Allen Test

test that determines the patency of the radial and ulnar arteries by compressing one artery site and observing return of skin color as evidence of patency of the other artery

Circumduction

the circular movement at the far end of a limb

Systole

the heart's pumping phase Contraction of the ventricles, known as emptying.

Rebound Tenderness

the production or intensification of pain when pressure that has been applied during palpation esp of the abdomen, is suddenly released

Arteriosclerosis

thickening and loss of elasticity of the arterial walls

Apex of Heart

tip of the heart pointing down toward the 5th left intercostal space Lower Portion of the Heart

Pericardium

tough, fibrous, double-walled sac that surrounds and protects the heart Heart's outermost layer

Eversion

turning the sole of the foot outward

Lesion GYRATE,

twisted, coiled spiral, snakelike

Palpitation

uncomfortable awareness of rapid or irregular heart rate

Arrhythmia

variation from the heart's normal rhythm

Profile Sign

viewing the finger from the side to detect early clubbing

Modify communication techniques as indicated by each patient's developmental stage, special needs, or cultural practices.

with older adults avoid "elder-speak" (similar to baby talk), face hearing impaired clients so they can read lips,

Relate anatomic structures to the correct landmark or anatomic location of the heart

1. the heart is roated so its right side is anterior and its left side is mostly posterior2.the right ventricle is immediately behind the sternum & forms the greatest area anterior cardiac surface3. the left ventricle lies behind the right ventricle & forms the apex & slender area of the left border4. The right atrium lies tot he right & above the right ventricle & forms the right boarder5. the left atrium is located posteriorly with only a small portion the left atrial appendage showing anteriorlyThe heart's position is more horizontal in an infant. The Apex is higher, located at 4th intercostal space. Reaches the adult position at 7 years of age.

Describe the function of the foramen ovale and the ductus arteriosus.

1.What is the ductus arteriosus and when does it close? Connects the aorta to the pulmonary artery, bypass the lungs, closes 10-15 hours after birth 2.What is the ductus venosus and when does it close? A shunt that allows oxygenated blood in the umbilical vein to bypass the liver, closes first week of life 3. What is the foramen ovale and when does it close? A small hole Located in the septum between the right and left atria. Closes at birth

1) Calculate Cardiac Output Remember: CO = SV x R Remember change to L/min a. Heart Rate= 185 bpm SV= 100 mL

18.5

Heart Assessment Pulmonic Area

2nd ICS at the Left Sternal Border (base of heart)

Aortic Area

2nd ICS at the Right Sternal Border (base of heart)

Erb's Point

3rd ICS near Left Midclavicular Line

Tricuspid Area

4th ICS at the Left Lower Sternal Border

Mitral (Apical)

5th ICS near the Left Midclavicualr Line (Apex of Heart)

Chalazion

A beady nodule protruding on the lid, chalazion is an obstruction and inflammation of a meibomian gland. If chronic, it is a nontender, firm, discrete swelling with freely movable skin overlying the nodule. If acutely inflamed, it is tender, warm, and red and points inside and not on lid margin (in contrast with stye).

Secondary Skin Lesions Keloid

A benign excess of scar tissue beyond sites of original injury: surgery, acne, ear piercing, tattoos, infections, burns.16 Looks smooth, rubbery, shiny and "clawlike"; feels smooth and firm. Found in ear lobes, back of neck, scalp, chest, and back; may occur months to years after initial trauma. Most common ages are 10-30 years; higher incidence in blacks, Hispanics, and Asians.

Venous Lake

A blue-purple dilation of venules and capillaries in a star-shaped, linear, or flaring pattern. Pressure causes them to empty or disappear. Located on the legs near varicose veins and also on the face, lips, ears, and chest.

Atopic Dermatitis (Eczema)

A chronic inflammatory skin lesion caused by overstimulated immune system, genetic changes in skin, and environmental triggers.16 Erythematous papules and vesicles, with weeping, oozing, flaking, fissures, crusts, and severe pruritus. Great effect on quality of life: sleep, behavior, mood, absences from school and work.

Pulse Deficit

A clinical sign wherein one is able to find a difference in count between heart beat and peripheral pulse. Occurs even as heart is contracting, can occur in atrial fibrilation, very early diastolic ventricular ectopic beats. Some patients with a pacemaker may exhibit this.

Mole (nevus)—

A clump of melanocytes, tan-to-brown color, flat or raised. Acquired nevi have symmetry, small size (6 mm or less), smooth borders, and single uniform pigmentation. The junctional nevus (Fig. 13.4, B) is macular only and occurs in children and adolescents. In young adults it progresses to the compound nevus (Fig. 13.4, C), which is macular and papular. The intradermal nevus (mainly in older age) has nevus cells in only the dermis.

Aphthous Ulcers

A common "canker sore" is a vesicle at first and then a small, round, "punched-out" ulcer with a white base surrounded by a red halo. It is quite painful and lasts for 1 to 2 weeks. The cause is unknown, although it is associated with stress, fatigue, and food allergy.

Branchial Remnant and Ear Deformity

A facial remnant or leftover of the embryologic branchial arch usually appears as a skin tag; in this case, one containing cartilage. Occurs most often in the preauricular area, in front of the tragus. When bilateral, there is increased risk for renal anomalies.

Spider or Star Angioma

A fiery red, star-shaped marking with a solid circular center. Capillary radiations extend from the central arterial body. With pressure, note a central pulsating body and blanching of extended legs. Develops on face, neck, or chest; may be associated with pregnancy, chronic liver disease, or estrogen therapy or may be normal.

Achondroplastic Dwarfism

A genetic disorder in converting cartilage to bone results in normal trunk size, short arms and legs, and short stature. It is characterized by a relatively large head with frontal bossing; midface hypoplasia (small); and often thoracic kyphosis, prominent lumbar lordosis, and abdominal protrusion. The mean adult height in men is about 131.5 cm (4 ft 4 in) and in women about 125 cm (4 ft 1 in).

Psoriasis

A hereditary chronic inflammatory skin disease with environmental triggers. Plaque psoriasis is a raised scaly, erythematous patch, with silvery scales, often pruritic and painful. Occurs on scalp, extensor surfaces of knees and elbows, lower back.5 Accompanied by nail pitting, onycholysis (see Table 13.12, p. 242).

Port-Wine Stain (Nevus Flammeus)

A large, flat, macular patch covering the scalp or face, frequently along the distribution of cranial nerve V. The color is dark red, bluish, or purplish and intensifies with crying, exertion, or exposure to heat or cold. The marking consists of mature capillaries. It is present at birth and usually does not fade. The use of yellow light lasers now makes photoablation of the lesion possible, with minimal adverse effects.

Scoliosis

A lateral S-shaped curvature of the thoracic and lumbar spine, usually with involved vertebrae rotation. Note unequal shoulder and scapular height and unequal hip levels, rib interspaces flared on convex side. More prevalent in adolescent age-groups, especially girls. Mild deformities are asymptomatic. If severe (>45 degrees) deviation is present, scoliosis may reduce lung volume, and person is at risk for impaired cardiopulmonary function. Primary impairment is cosmetic deformity, negatively affecting self-image.

Enophthalmos (Sunken Eyes)

A look of narrowed palpebral fissures shows with enophthalmos, in which the eyeballs are recessed. Bilateral enophthalmos is caused by loss of fat in the orbits and occurs with dehydration and chronic wasting illnesses. For illustration, see Cachectic Appearance in Table 14.5, p. 273.

Pectus Excavatum

A markedly sunken sternum and adjacent cartilages (also called funnel breast). Depression begins at second intercostal space, becoming depressed most at junction of xiphoid with body of sternum. More noticeable on inspiration. Congenital, usually not symptomatic. When severe, sternal depression may cause embarrassment and a negative self-concept. Surgery may be indicated.

Ingrown Toenail

A misnomer; the nail does not grow in, but the soft tissue grows over the nail and obliterates the groove. It occurs almost always on the great toe on the medial or lateral side. It is caused by trimming the nail too short or toe crowding in tight shoes. The area becomes infected when the nail grows and its corner penetrates the soft tissue.

Thrill

A palpable vibration that signifies turbulent blood flow. (Feel)

Ecchymosis

A purplish patch resulting from extravasation of blood into the skin, >3 mm in diameter.

Strawberry Mark (Immature Hemangioma)

A raised bright red area with well-defined borders about 2 to 3 cm in diameter. It does not blanch with pressure. It consists of immature capillaries, is present at birth or develops in the first few months, and usually disappears by age 5 to 7 years. Requires no treatment, although parental and peer pressure may prompt treatment.

Subconjunctival Hemorrhage

A red patch on the sclera, subconjunctival hemorrhage looks alarming but is usually not serious. The red patch has sharp edges like a spot of paint, although here it is extensive. It occurs from increased intraocular pressure from coughing, vomiting, weight lifting, labor during childbirth, straining at stool, or trauma.

Cavernous Hemangioma (Mature)

A reddish-blue, irregularly shaped, solid and spongy mass of blood vessels. It may be present at birth, may enlarge during the first 10 to 15 months, and does not involute spontaneously.

Retention "Cyst" (Mucocele)

A round, well-defined, translucent nodule that may be very small or up to 1 to 2 cm. It is a pocket of mucus that forms when a duct of a minor salivary gland ruptures. The benign lesion also may occur on the buccal mucosa, on the floor of the mouth, or under the tip of the tongue.

Magenta Tongue

A sign of riboflavin deficiency. In contrast, a pale tongue is probably attributable to iron deficiency; a beefy red-colored tongue is caused by vitamin B-complex deficiency.

Tooth Avulsion

A traumatic injury may dislodge a primary (deciduous) or a permanent tooth from its alveolar socket. Trauma is often the result of falls or sports collision. The time to reimplantation is crucial for viability. During this time, the tooth must be stored in an appropriate solution (milk is acceptable and at hand) because dry storage of >15 minutes increases the risk of necrosis. Do not touch root; ask patient about tetanus vaccine and need for bacterial endocarditis prophylaxis.4

Candidiasis or Monilial Infection

A white, cheesy, curdlike patch on the buccal mucosa and tongue. It scrapes off, leaving a raw, red surface that bleeds easily. Termed thrush in the newborn. It is an opportunistic infection that occurs after the use of antibiotics and corticosteroids and in immunosuppressed people.

1) Before collecting ABG's from the radial artery a _________ _________ test will be performed. (Only use second word for lock)

ALLEN

Ankylosing Spondylitis (AS)

AS is chronic inflamed vertebrae (spondylitis) that in extreme form leads to bony fusion of vertebral joints (ankyloses). It affects the spine, pelvis, and thoracic cage, and is characterized by inflammatory back pain that is dull and deep in lower back or buttocks.31 It also has morning back stiffness that lasts ≥30 minutes and decreases with activity, nighttime awakening with pain, age at onset ≤45 years. It affects males by a 2 : 1 ratio, beginning in late adolescence or early 20s. Spasm of paraspinal muscles pulls spine into forward flexion, obliterating cervical and lumbar curves. Thoracic curve exaggerated into single kyphotic rounding. Also includes flexion deformities of hips and knees as they compensate for spinal flexion.

Primary Angle-Closure Glaucoma (PACG)

Acute narrow-angle glaucoma shows circumcorneal redness around the iris, with a dilated pupil. Pupil is oval, dilated; cornea looks "steamy"; and anterior chamber is shallow. Acute glaucoma occurs with sudden increase in intraocular pressure from blocked outflow from anterior chamber. The person experiences a sudden clouding of vision, sudden eye pain, and halos around lights. This requires emergency treatment to avoid permanent vision loss.12

Describe the characteristics of heart sounds

All heart sounds are described by: 1. Frequency (pitch)—Heart sounds are described as high pitched or low pitched, although these terms are relative because all are low-frequency sounds, and you need a good stethoscope to hear them. 2. Intensity (loudness)—Loud or soft 3. Duration—Very short for heart sounds; silent periods are longer 4. Timing—Systole or diastole Dull sounds are heard with the diaphragm of the stethoscope, high pitched sounds are heard with the bell. Most normal sounds are heard with the diaphragm.S1 & S2 is the lub-dub S2- S4-

Upward Palpebral Slant

Although normal in many children, when combined with epicanthal folds, hypertelorism (large spacing between the eyes), and Brushfield spots (light-colored areas in outer iris), it indicates Down syndrome.

Stroke Volume

Amount of blood pumped out of the heart with each heartbeat

Kyphosis

An exaggerated posterior curvature of the thoracic spine (humpback) that causes significant back pain and limited mobility. Severe deformities impair cardiopulmonary function. If the neck muscles are strong, compensation occurs by hyperextension of head to maintain level of vision. Kyphosis is associated with aging, especially the "dowager's hump" of postmenopausal osteoporotic women. However, it is common well before menopause. Women with adequate exercise habits are less likely to have kyphosis.

Otitis Externa (Swimmer's Ear)

An infection of the outer ear, with severe painful movement of the pinna and tragus, redness and swelling of pinna and canal, scanty purulent discharge, scaling, itching, fever, and enlarged tender regional lymph nodes. Hearing normal or slightly diminished. More common in hot, humid weather. Swimming causes canal to become waterlogged and swell; skinfolds set up for infection. Prevent by using rubbing alcohol or 2% acetic acid eardrops after every swim.

Deviation in Nipple Pointing

An underlying cancer causes fibrosis in the mammary ducts, which pulls the nipple angle toward it. Here note the swelling behind the right nipple and that the nipple tilts laterally.

Common Sites of Referred Abdominal Pain

§Liver—RUQ §Esophagus—behind lower sternum §Ulcer—shoulder §Gallbladder—RUQ §Appendix—RLQ §Pancreas—Midscapular §Kidney—flank pain §Small intestine—diffuse §Colon—colicky pain and bloating Liver. Hepatitis may have mild-to-moderate dull pain in right upper quadrant (RUQ) or epigastrium, along with anorexia, nausea, malaise, low-grade fever. Esophagus. Gastroesophageal reflux disease (GERD) is a complex of symptoms of esophagitis, including burning pain in midepigastrium or behind lower sternum that radiates upward, or "heartburn." Occurs 30 to 60 minutes after eating; aggravated by lying down or bending over. Gallbladder. Cholecystitis is biliary colic, sudden pain in RUQ that may radiate to right or left scapula and that builds over time, lasting 2 to 4 hours, after ingestion of fatty foods, alcohol, or caffeine. Associated with nausea and vomiting and with positive Murphy sign or sudden stop in inspiration with RUQ palpation. Pancreas. Pancreatitis has acute, boring midepigastric pain radiating to the back and sometimes to the left scapula or flank, severe nausea, and vomiting. Duodenum. Duodenal ulcer typically has dull, aching, gnawing pain; does not radiate; may be relieved by food; and may awaken the person from sleep. Stomach. Gastric ulcer pain is dull, aching, gnawing epigastric pain, usually brought on by food and radiates to back or substernal area. Pain of perforated ulcer is burning epigastric pain of sudden onset that refers to one or both shoulders. Appendix. Appendicitis typically starts as dull, diffuse pain in periumbilical region that later shifts to severe, sharp, persistent pain and tenderness localized in RLQ (McBurney point). Pain is aggravated by movement, coughing, deep breathing; associated with anorexia, then nausea and vomiting, fever. Kidney. Kidney stones prompt a sudden onset of severe, colicky flank or lower abdominal pain. Small intestine. Gastroenteritis has diffuse, generalized abdominal pain with nausea, diarrhea. Colon. Large bowel obstruction has moderate, colicky pain of gradual onset in lower abdomen and bloating. Irritable bowel syndrome (IBS) has sharp or burning cramping pain over a wide area; does not radiate. Brought on by meals; relieved by bowel movement.

(((((((((((((((((((((()))))))))))))))))))) Identify the organs located within each of the four abdominal quadrants.

§Right upper quadrant (RUQ) - Liver -Gallbladder - Duodenum - Head of pancreas - Right kidney and adrenal gland - Hepatic flexure of colon - Part of ascending and transverse colon §Left upper quadrant (LUQ) - Stomach - Spleen - Left lobe of liver - Body of pancreas - Left kidney and adrenal gland -Splenic flexure of colon - Part of transverse and descending colon §Right lower quadrant (RLQ) - Cecum - Appendix - Right ovary and tube - Right ureter - Right spermatic cord §Left lower quadrant (LLQ) - Part of descending colon - Sigmoid colon - Left ovary and tube - Left ureter - Left spermatic cord

Differentiate among the types of hearing loss.

´Conductive hearing loss: mechanical dysfunction of external or middle ear ´Partial loss because a person is able to hear if sound amplitude is increased enough to reach normal nerve elements in inner ear ´May be caused by impacted cerumen, foreign bodies, a perforated TM, pus or serum in middle ear, and otosclerosis ´Sensorineural (or perceptive) hearing loss: signifies pathology of inner ear, cranial nerve VIII, or auditory areas of cerebral cortex ´Increase in amplitude may not enable a person to understand words. ´May be caused by presbycusis and by ototoxic drugs, which affect hair cells in cochlea. ´Mixed hearing loss: combination of conductive and sensorineural types in the same ear

What is a... Fibrous Joint? Example?

•Bones are united by interjacent fibrous tissue or cartilage and are immovable Sutures Skull, Syndesmoses (between long bones) radial/ulna, tibia/fibula

•What is osteoporosis? •Who is most likely to be affected? Age, Gender, Ethnicity •What are some preventative measures?

•Disease involving the loss of mineralized bone mass and leading to porous bone and thus risk of fractures. •Older Age, Postmenopausal Caucasian Women •Physical Exercise- Fast walking best prevention for Osteoporosis •Smoking Cessation, Reduced Alcohol Intake •Diet •Dark green and deep yellow vegetables (spinach, romaine lettuce, etc.) •Vitamin D and Calcium for bone health

What is a •Synovial Joint? Example?

•Freely moveable because their bones are separated from one another and enclosed in a joint cavity. Joint cavity is lined with synovial membrane that secretes lubrication or synovial fluid. •Pivot Joint (C1 and C2 vertebrae), Hinge Joint (Elbow), Saddle Joint (Hand), Plane Joint (foot/between tarsals), Condyloid joint (wrist), Ball and Socket (Hip)

Discuss developmental care associated with the thoracic cavity.

•Infants and children •Development in utero with birth demanding instant performance •Increased vulnerability of respiratory system: Associated with environmental tobacco smoke (ETS) exposure include •sudden infant death syndrome, negative behavioral and cognitive functioning, and increased rates of adolescent smoking. •Pregnant woman •Impact of enlarging uterus •Impact of physiologic dyspnea •Aging adult •Decreased vital capacity and increased residual volume based on structural changes •Histologic changes lead to decreased gas exchange.

Identify the structures and function of the regional lymphatics.

•Major part of immune system • Detects and eliminates foreign substances from body •Rich supply of lymph nodes •Greatest supply is in head and neck. •Lymphatic drainage •Helps to prevent potentially harmful substances from entering the circulation •You should be familiar with direction of drainage patterns of lymph nodes. •

•What is Barlow and Ortolani Tests?

•Manuvears Performed together •The Ortalani test relocates the disclocation of the hip joint that has just been elicited by the Barlow Test •Done at every visit until 1 years old •Checking for developmental dysplasia of the hip (DDH)

Describe various routes of temperature measurement and special considerations for each route.

•Oral temperature accurate and convenient •Oral sublingual site has rich blood supply from carotid arteries that quickly responds to changes in inner core temperature. •Normal oral temperature in a resting person is37° C (98.6° F), with a range of 35.8° C to 37.3° C (96.4° F to 99.1° F) •Rectal measures 0.4° C to 0.5° C (0.7° F to 1° F) higher •Tympanic •Temporal Artery Thermometer - Oral: accurate and convenient for this old enough for oral control - Rectal: good for those who cannot close their mouth but not many others for patient discomfort - Tympanic Membrane: speed, convenience, safety, and noninvasiveness - Temporal Artery: take multiple readings for accuracy

Define pupillary light reflex, fixation, and accommodation.

•Pupillary light reflex: normal constriction of pupils when bright light shines on retina •Subcortical reflex arc with no conscious control •Fixation: a reflex direction of eye toward an object attracting a person's attention •Image fixed in center of visual field, the fovea centralis •Accommodation: adaptation of eye for near vision •Accomplished by increasing curvature of lens through movement of ciliary muscles •Although lens cannot be observed directly, the following components of accommodation can be observed: •Convergence (motion toward) of the axes of the eyeballs •Pupillary constriction

Changes in the Abdomen with Aging

•Salivation Decreases- Dry Mouth, Decreased Taste •Esophageal Emptying Delayed- Risk for Aspiration •Gastric Acid Secretion Decreases (Anemias) •Gallstone incidences increase •Liver size decreases •Drug metabolism impaired, increased side effects •Constipation (Not a physiologic consequence) •

What is a... Cartilaginous Joint? Example?

•Separated by fibrocartilaginous discs and are only slightly moveable •Vertebrae, Between Manubrium/Sternum, pubic symphysis

Define different types of pain.

•Visceral pain originates from larger interior organs. •Stems from direct injury to organ or from stretching of organ from tumor, ischemia, distention, or severe contraction •Presents with autonomic responses such as vomiting, nausea, pallor, and diaphoresis •Deep somatic pain comes from sources such as blood vessels, joints, tendons, muscles, and bone. •Injury may result from pressure, trauma, or ischemia. •Described as aching or throbbing •Like visceral pain it can be accompanied by nausea, sweating, tachycardia, and HTN. •Acute pain •Short-term and self-limiting: •Often follows a predictable trajectory, and dissipates after an injury heals •Self-protective purpose: •Acute pain warns individual of actual or potential tissue damage. •Incident pain: •Type of acute pain that occurs predictably with certain movements •Chronic pain can be further divided into malignant (cancer related) and nonmalignant. •In contrast, chronic (or persistent) pain is diagnosed when pain continues for 6 months or longer. •Chronic nonmalignant pain is often associated with: •musculoskeletal conditions, such as arthritis, low back pain, or fibromyalgia •Unfortunately, many patients with chronic pain are not believed. •Often labeled as attention seekers, drug seekers, and so forth •Breakthrough pain •Transient spike in pain level with moderate to severe intensity in an otherwise controlled pain syndrome •Can result from: •End of dose medication failure •Result of incident or episodic pain •Treatment: •Shorten interval dosing and/or increase medication •Experience of pain is a complex biopsychosocial mechanism. •More clinical research is needed. •Rely on patient report as best indicator of pain •

Mammograms

•What age should a female have their first mammogram? •Age 40-44, could be sooner based on risk factors for baseline •What is the recommendation for how often? •Every two years per CDC, however if increased risk annually •Where are breast tumors usually found? Which quadrant? •Upper axillary quadrant •What is BRCA? •BReast CAncer Gene- increased risk of breast cancer •Mammograms •What age should a female have their first mammogram? •Age 40-44, could be sooner based on risk factors for baseline •What is the recommendation for how often? •Every two years per CDC, however if increased risk annually •Where are breast tumors usually found? Which quadrant? •Upper axillary quadrant •What is BRCA? •BReast CAncer Gene- increased risk of breast cancer •

Describe the significant differences between the skin of the very young, the older adult, and the healthy adult.

■Newborn infants -Lanugo: fine downy hair of newborn infant -Vernix caseosa: thick, cheesy substance -Sebum: holding water in the skin producing milia ■Children -Epidermis thickens, darkens, and becomes lubricated. -Hair growth accelerates. ■Adolescents -Secretions from apocrine sweat glands increase. -Subcutaneous fat deposits increase. -Secondary sex characteristics pregnant woman ■Increase in metabolism leads to increase secretion of sweat and sebaceous glands to dissipate heat. ■Expected skin color changes due to increased hormone levels. ■Fat deposits are laid down as maternal reserves for nursing baby. aging adults ■Elasticity -Loses elasticity; skinfolds and sags ■Sweat and sebaceous glands -Decrease in number and function, leaving skin dry ■Senile purpura -Discoloration due to increasing capillary fragility ■Skin breakdown due to multiple factors -Cell replacement is slower and wound healing is delayed. ■Hair matrix -Functioning melanocytes decrease, leading to gray fine hair ■

Relate the anatomic structures of the skin to its functions.

■Skin is waterproof, protective, and adaptive -Protection from environment -Prevents penetration -Perception -Temperature regulation -Identification -Communication -Wound repair -Absorption and excretion -Production of vitamin D Functions of the skin include protection, prevention of penetration, perception (of touch, pain, temperature, and pressure), temperature regulation, identification, communication, wound repair, absorption and excretion, and production of vitamin D. The skin produces vitamin D, not vitamin C. The basal cell layer of the epidermis forms new skin cells. Melanocytes produce melanin, which gives brown tones to the skin and hair. Sebum is produced by the sebaceous glands to lubricate the skin and hair.

Recognize health care professionals' role as mandatory reporters of abuse.

◦Four main categories ◦Physical violence: force resulting in injury or death ◦Sexual violence: attempted or completed acts without permission ◦Stalking: repeated unwanted attention through various methods ◦Psychological aggression: emotional abuse of an aggressive nature ◦Also includes teen dating violence ◦Can be physical, sexual, psychological, or emotional ◦Youths who experience this are more likely to experience mental health issues and/or participate in unhealthy behaviors. ◦Sexting or cyber abuse can be a means of access. Child Abuse and Neglect ◦Defined at state and federal levels—The Child Abuse Prevention and Treatment Act ◦Recently amended to include sex and human trafficking ◦Enhance protection for infants with Fetal Alcohol Spectrum Disorder ◦General definitions ◦Neglect: failure to provide for children's basic needs ◦Physical abuse: nonaccidental injury that leads to harm of a child ◦Sexual abuse: fondling, sexual acts, exploitation, and trafficking ◦Emotional abuse: pattern of behavior that harms a child's sense of self-worth or development


Related study sets

c-17 and c-18 development theory stuff

View Set

Greek Contributions - Mythology, Art, Science, Philosophy

View Set

الادب / مدرسة الاحياء و البعث

View Set

Microbiology Chapter 16: Food and Industrial Microbiology

View Set

Pediatric Success Respiratory Disorders

View Set

Risk Management: Managing Life Cycle Risks

View Set